Google Groups no longer supports new Usenet posts or subscriptions. Historical content remains viewable.
Dismiss

particle spin

1 view
Skip to first unread message

KenMagel

unread,
Dec 25, 2003, 2:57:44 PM12/25/03
to
If a particle were completely at rest it would still have momentum as a result
of quantum spin. If the spin of the proton could be stopped an the momentum
transfered to velocity how fast would the proton be traveling. How fast does
the surface of a proton rotate as a result of quantum spin?

Jet Red

unread,
Dec 25, 2003, 5:51:57 PM12/25/03
to
You are applying classical constructs to understand quantum behaviors...
spin is meaningless as a linear momentum/velocity construct... the proton
has no "surface" and doesn't rotate in the classical sense... spin is a
degree of freedom dictated by relativistic qm that is associated with a
internal angular momentum...

"KenMagel" <kenm...@aol.com> wrote in message
news:20031225145744...@mb-m19.aol.com...

Andr? Michaud

unread,
Dec 26, 2003, 7:04:53 PM12/26/03
to
kenm...@aol.com (KenMagel) wrote in message news:<20031225145744...@mb-m19.aol.com>...

Protons are not elementary particles. They are a system made up of
2 up quarks and 1 down quark locked into an intricate dynamic system.
Consequently, protons can have no surface, just like the Solar
System can have no surface.

Proton spin is the resultant of the relative spin state of the 3
constituting quarks.

Fundamentally, spin is a reflection of the relative magnetic orientation
of a particle with respect to other particles.

For example, the covalent bond between atoms is made up of two electrons
associated by antiparallel spin.

@tshankha Dave Thomson

unread,
Dec 26, 2003, 9:49:11 PM12/26/03
to
"KenMagel" <kenm...@aol.com> wrote in message
news:20031225145744...@mb-m19.aol.com...

In the Standard Model spin is abstract, and aside for carrying a fraction,
is meaningless.

However, I have used the empirical data from the Standard Model to produce a
different system of physics. In this new system, the proton and electron
are true quantum particles. These particles do have a surface and do spin
at the speed of light.

The spin is imparted by the Aether, which itself has a quantum unit equal to
rotating magnetic field. The advantage of my system of physics is that it
properly identifies all charge as distributed and produces a very simple
Unified Field Theory.
www.tshankha.com

Dave


John Devers

unread,
Dec 27, 2003, 9:53:39 AM12/27/03
to
s...@microtec.net (Andr? Michaud) wrote in message news:<562f286c.03122...@posting.google.com>...


>
> Proton spin is the resultant of the relative spin state of the 3
> constituting quarks.
>


Bringing the Nucleon into Sharper Focus


http://www.aip.org/enews/physnews/2003/split/666-1.html


http://www.aip.org/mgr/png/images/quark-spin-png.jpg

In experiments at Jefferson National Accelerator Facility in Virginia,
a multinational research team has found that the spins of the proton's
two up quarks (u) are aligned parallel to the overall spin of the
proton, but the same is not true for the proton's down quark (d).
In order to make the experimental data on quark spin agree with
theory, researchers had to take into account the once-neglected
orbital motions of quarks inside the proton. Therefore, these
experiments provide many new details on the behavior of quarks in the
proton.

Yousuf Khan

unread,
Dec 27, 2003, 1:19:07 PM12/27/03
to
"John Devers" <johnd...@iprimus.com.au> wrote in message
news:6f838e26.03122...@posting.google.com...

> Bringing the Nucleon into Sharper Focus
>
>
> http://www.aip.org/enews/physnews/2003/split/666-1.html

The article mentioned that nucleons are surrounded by a sea of
quark-antiquark pairs that pop in and out of existence randomly (i.e. sea
quarks). I know these sort of things happen normally in empty space anyways.
Are they suggesting that it happens with more probability near the quarks of
a nucleon? If so, what's the purpose of them?

Here's the quote:
"Aiming an electron beam at a helium-3 target in JLab's Hall A, researchers
(led by Jian-Ping Chen, jpc...@jlab.org and Zein-Eddine Meziani,
mez...@temple.edu) selected a 5.7 GeV beam energy so that the electrons
interacted mainly with the neutron's valence quarks and not its sea quarks
and gluons. "

Yousuf Khan


Andr? Michaud

unread,
Dec 28, 2003, 3:59:52 PM12/28/03
to
"Yousuf Khan" <removethisspam.bj...@hotmail.com> wrote in message news:<v2kHb.178075$ea%.30194@news01.bloor.is.net.cable.rogers.com>...

> "John Devers" <johnd...@iprimus.com.au> wrote in message
> news:6f838e26.03122...@posting.google.com...
> > Bringing the Nucleon into Sharper Focus
> >
> >
> > http://www.aip.org/enews/physnews/2003/split/666-1.html
>
> The article mentioned that nucleons are surrounded by a sea of
> quark-antiquark pairs that pop in and out of existence randomly (i.e. sea
> quarks). I know these sort of things happen normally in empty space anyways.

Well... theoretically only. This notion is a postulate of QFT (Quantum Field
Theory). Only a theory.

In physical reality, the existence or non-existence of such a sea of
complementary pairs so conveniently popping randomly in and out of
existence has never been convincingly proven, nor can it be apparently.

So this simply boils down to a convenient mathematical tool.

> Are they suggesting that it happens with more probability near the quarks
> of a nucleon?

It may look like it from the article, but it is only a reflection of
their personal belief in their existence.

Presently, there was no chance at all that the beam interfered with
the so called "sea of quarks and gluons". Such interference with
"mathematical concepts" would in fact have been a much greater event
than the discovery that the motion of the very real valence quarks
inside nucleons needed to be taken into account.

> If so, what's the purpose of them?

Simple testimony of their faith in the value of QFT.



> Here's the quote:
> "Aiming an electron beam at a helium-3 target in JLab's Hall A, researchers
> (led by Jian-Ping Chen, jpc...@jlab.org and Zein-Eddine Meziani,
> mez...@temple.edu) selected a 5.7 GeV beam energy so that the electrons
> interacted mainly with the neutron's valence quarks and not its sea quarks
> and gluons. "

André Michaud

Yousuf Khan

unread,
Dec 29, 2003, 1:19:57 AM12/29/03
to
"Andr? Michaud" <s...@microtec.net> wrote in message
news:562f286c.03122...@posting.google.com...

> > Are they suggesting that it happens with more probability near the
quarks
> > of a nucleon?
>
> It may look like it from the article, but it is only a reflection of
> their personal belief in their existence.
>
> Presently, there was no chance at all that the beam interfered with
> the so called "sea of quarks and gluons". Such interference with
> "mathematical concepts" would in fact have been a much greater event
> than the discovery that the motion of the very real valence quarks
> inside nucleons needed to be taken into account.

So if they wanted a Nobel, then they should've actually been trying to
reduce the power in their accelerator beam, in order to try and interfere
with such mathematical concepts? :-)

Yousuf Khan


Y.Porat

unread,
Dec 29, 2003, 3:15:49 AM12/29/03
to
s...@microtec.net (Andr? Michaud) wrote in message news:<562f286c.03122...@posting.google.com>...
-----------------
well saied Andre
yet i think you are too gentle
being too gentle will not do the right thing to scince
and with the proper pace.
i would put it much more blantly:
( a shock is needed the all that parrot comunity of establishment scince)
so
the 'sea of quarks' is nothing but *dead on arrival*
it is a nonstarter!
you dont need to wait for expensive exoeriments to realise it
you cannot find in experiments only less than 10 percent of
quarks and 'fill it ' with another 90 percent of 'Glueons'

it does not make 'physics sense'
the other word for glueons is *unknown*
yet people forgot that 'glueons' stands nothing but for 'unknown'
(not to mention vain ignorance)
if you want it more blantly - it is cheating!
(not to mention the huge resourses waist that is to come with it.)
----------
Regards
Y.Porat
-----------------

Andr? Michaud

unread,
Dec 29, 2003, 8:27:39 AM12/29/03
to
"Yousuf Khan" <bbbl6...@yahoo.com.nospam> wrote in message news:<hIPHb.225447$%TO.9...@twister01.bloor.is.net.cable.rogers.com>...

They certainly would get a Nobel for such a feat. But lowering the
energy of the beam would simply make it not energetic enough to
penetrate inside protons. The energy level needed was determined
experimentally in 1968 at SLAC when they first verified the
physical existence of up and down quarks in the same manner.

André Michaud.

Andr? Michaud

unread,
Dec 29, 2003, 12:59:48 PM12/29/03
to
map...@012.net.il (Y.Porat) wrote in message news:<4e35159f.03122...@posting.google.com>...

Just trying to present things objectively, Yehiel.

I think that material presented objectively is more likely to
be considered for verification by young readers than agressive
strikes.

> i would put it much more blantly:
> ( a shock is needed the all that parrot comunity of establishment
> scince) so the 'sea of quarks' is nothing but *dead on arrival*
> it is a nonstarter!

Unfortunately, no amount of shock has ever convinced past physical
communities of the need to reconsider premises.

Planck summarized this problem very precisely in the early 1900's:

"A new scientific truth doesn't triumph by convincing it opponents
and making them see the light, but rather because its opponents end
up dying, and the following generation grows, becoming familiar
with it."
Max Planck, 1900

You can rest assured that nothing has changed. As has always happened
in the past, current establishment will pass away without having
even considered reconsidering.

Only individuals from the up coming generation will even consider
requestioning fundamental premises, as they always do for some time
before setting their mind for good. The only condition for change to
catch on is that the proper information be made available to them
before they set their minds irreversibly to established orthodox
thinking.

So, anyone who attempts to "shock" the established physical
community into reconsidering is bound to fail. The only "shockable"
minds around are those of young readers who have not yet made up
their minds as to what they believe is the right way to go.

> you dont need to wait for expensive exoeriments to realise it
> you cannot find in experiments only less than 10 percent of
> quarks and 'fill it ' with another 90 percent of 'Glueons'
>
> it does not make 'physics sense'

Agreement.

> the other word for glueons is *unknown*

Absolutely right. Gluons were imagined to represent in the formulas of
QCD (Quantum Chromodynamics) the force not yet understood that binds
real quarks inside nucleons. The same goes for quarks other than up and
down. All virtual stuff.

But even after 30 years of existence, no one has been able to formulate
the equations of QCD with sufficient precision for it to correctly
describe a nucleon.

Another failed theory that is still considered and taught as current
simply because it is the best that can be cooked up based on current
premises.

> yet people forgot that 'glueons' stands nothing but for 'unknown'
> (not to mention vain ignorance)

True that many physicists believe in their existence. But most
rationally know that they are mathematical concepts, to be used
for lack of better tools to do the job.

> if you want it more blantly - it is cheating!

Well, maybe not. I think that physicists are honest. They simply
deeply believe that the set of fundamental premises that underlies
orthodox theories is not to be questioned.

A sad restraint to progress.

> (not to mention the huge resourses waist that is to come with it.)

Total agreement. If only a fraction of that money was used for
real fundamental reconsideration, progress would rapidly come.

But I don't see how the grant system can be changed to serve more
than individuals' selfish interests.

Regards

André Michaud

Y.Porat

unread,
Dec 30, 2003, 3:53:56 AM12/30/03
to
s...@microtec.net (Andr? Michaud) wrote in message news:<562f286c.03122...@posting.google.com>...
> map...@012.net.il (Y.Porat) wrote in message news:<4e35159f.03122...@posting.google.com>...
> > s...@microtec.net (Andr? Michaud) wrote in message news:<562f286c.03122...@posting.google.com>...
> > > "Yousuf Khan" <removethisspam.bj...@hotmail.com> wrote in message news:<v2kHb.178075$ea%.30194@news01.bloor.is.net.cable.rogers.com>...
> > > > "John Devers" <johnd...@iprimus.com.au> wrote in message
> > > > news:6f838e26.03122...@posting.google.com...
> > > > > Bringing the Nucleon into Sharper Focus
> > > > >
> > > > >
> > > > > http://www.aip.org/enews/physnews/2003/split/666-1.html
> > > >
> Hi Andre
sorry to top post but it is too long:
we fully agree!! so .....
you see that our battle is not a lost battle!
i am not sure about your presentation of *all scintists*
as honest..
you forgot the 'economic aspect of it ' (keeping jobs etc)
so if there are at least two people who agree, (about the above)
it is probably only the 'tip of the iceberg'-
there are a lot more than just the two of us.
Happy new year
Y.Porat
----------------

> > >
> > > Presently, there was no chance at all that the beam interfered with
> > > the so called "sea of quarks and gluons". Such interference with
> > > "mathematical concepts" would in fact have been a much greater event
> > > than the discovery that the motion of the very real valence quarks
> > > inside nucleons needed to be taken into account.
> > >
> > > > If so, what's the purpose of them?
> > >
> > > Simple testimony of their faith in the value of QFT.
>
> > >
> > > André Michaud
> > -----------------

Andr? Michaud

unread,
Dec 30, 2003, 12:52:48 PM12/30/03
to
map...@012.net.il (Y.Porat) wrote in message

> Hi Andre


> sorry to top post but it is too long:
> we fully agree!! so .....
> you see that our battle is not a lost battle!
> i am not sure about your presentation of *all scintists*
> as honest..

I was specifically referring to their honesty regarding their
deep belief in the set of fundamental premises that is recognized
("imposed as fundamental truth", would be more to the point)
by the orthodox community.

> you forgot the 'economic aspect of it ' (keeping jobs etc)

Did I !

I was addressing that specific problem with my last comment on the
previous post:

"> > But I don't see how the grant system can be changed to serve more
> > than individuals' selfish interests."

> so if there are at least two people who agree, (about the above)


> it is probably only the 'tip of the iceberg'-
> there are a lot more than just the two of us.

You can bet on it. I know many personally.

The Internet has forever change the hand. To the frustration of many
physicists who consider deep knowledge on fundamental physics their
exclusive preserve, that knowledge is now available to all who care
to explore and learn.

The establishment is now unable, as it could before, to prevent anyone
from learning and exploring on their own. They have totally lost control,
but most are unaware of it. Their preserve is no longer fenced in.

The long unfortunate 70 years+ lull in fundamental research that was
imposed by the Copenhagen school of thought is about to end. I estimate
that within 10 to 15 years, fundamental research unrestrained by dogma
will have resumed.

Despite the immediacy of info communication on the net, books remain
the media that has the most staying power.

Like articles in magazines, most internet sites are quickly visited
and soon forgotten, the profusion of information leaves no time to
deeply integrate and verify info.

This does not occur with books. When someone picks up a book with the
intent of understanding the subject. That person is more likely to
invest the time and mental effort to really grasp the concepts described.

That's the way I personally chose. I have unrestrained access to physics
labs in schools of all levels and I have been sowing my own brand of
seeds of copenhagen demise for 3 years now. Thousands of copies have
now been dispersed. There is no way that all of them will be ignored.

An added nudge (hopefully efficient) to the valuable efforts of others.

> Happy new year
> Y.Porat

Happy new years and long health

André Michaud

Y.Porat

unread,
Dec 31, 2003, 4:11:13 AM12/31/03
to
s...@microtec.net (Andr? Michaud) wrote in message news:<562f286c.03123...@posting.google.com>...

> map...@012.net.il (Y.Porat) wrote in message
>
> > Hi Andre
> > sorry to top post but it is too long:
> > we fully agree!! so .....
> > you see that our battle is not a lost battle!
> > i am not sure about your presentation of *all scintists*
> > as honest..
>
>
>
--------------
well saied Andre
you say it much better than me
Y.Porat
-----------------

Andr? Michaud

unread,
Jan 1, 2004, 4:47:59 PM1/1/04
to
"Dave Thomson" <news3 @ tshankha<no spam>.com> wrote in message news:<Tt6Hb.43$jH5....@news.uswest.net>...

It is a verified fact that protons are not elementary particles,
quantum or otherwise. As to the speed of the 3 constituting quarks,
it is effectively quite likely that they may be locked in dynamic
local close circuit motion at very nearly the speed of light but no
amount of tweaking of reality can cause the proton itself to have
a surface.

It is also not really possible for the electron to have a surface.

However logical your new system may be, if it is not grounded in verified
facts about particles, it is bound to remain further away from a true
representation of reality than any system that does.

André Michaud

@tshankha.nospam.com Dave Thomson

unread,
Jan 2, 2004, 11:51:34 AM1/2/04
to
"Andr? Michaud" <s...@microtec.net> wrote in message
news:562f286c.04010...@posting.google.com...

> It is a verified fact that protons are not elementary particles,
> quantum or otherwise. As to the speed of the 3 constituting quarks,
> it is effectively quite likely that they may be locked in dynamic
> local close circuit motion at very nearly the speed of light but no
> amount of tweaking of reality can cause the proton itself to have
> a surface.

It is not verified that protons are not quantum particles. If you're saying
quarks are smaller particles than protons, then give me the mass and charge
of each right here. If the quark theory is correct, and the proton is made
of 3 of them, then at least one of the quarks must have a mass greater than
600 times the electron. What is the mass of this particle?

> It is also not really possible for the electron to have a surface.

The charge has an effective surface. It is not a surface of length, but a
distributed charge. The mass is more like a string.

> However logical your new system may be, if it is not grounded in verified
> facts about particles, it is bound to remain further away from a true
> representation of reality than any system that does.

My theory is completely grounded in empirical data. The dimensions are
exactly as measured and listed at NIST.

If you are not fully instructed in the Quantum Physics Model, but try to
compare the concepts of the Quantum Physics Model to the concepts of the
Standard Model, you will only get confused. There is not a one to one
relation between the theories. The Quantum Physics Model is a completely
different structure for the same exact empirical data used for the structure
of the Standard Model. The data is the same, but the way it is viewed is
different. You need to learn a completely different paradigm to see the
logic of the Quantum Physics Model.

Dave


Yousuf Khan

unread,
Jan 2, 2004, 1:11:39 PM1/2/04
to
"Dave Thomson" <news3 @ tshankha.no spam.com> wrote in message
news:DnhJb.915$Gn2....@news.uswest.net...

> It is not verified that protons are not quantum particles. If you're
saying
> quarks are smaller particles than protons, then give me the mass and
charge
> of each right here. If the quark theory is correct, and the proton is
made
> of 3 of them, then at least one of the quarks must have a mass greater
than
> 600 times the electron. What is the mass of this particle?

They say most of the mass is tied up not in the quarks, but in the gluons.

Yousuf Khan


Andr? Michaud

unread,
Jan 2, 2004, 6:00:39 PM1/2/04
to
"Dave Thomson" <news3 @ tshankha.no spam.com> wrote in message news:<DnhJb.915$Gn2....@news.uswest.net>...
> "Andr? Michaud" <s...@microtec.net> wrote in message
> news:562f286c.04010...@posting.google.com...
> > It is a verified fact that protons are not elementary particles,
> > quantum or otherwise. As to the speed of the 3 constituting quarks,
> > it is effectively quite likely that they may be locked in dynamic
> > local close circuit motion at very nearly the speed of light but no
> > amount of tweaking of reality can cause the proton itself to have
> > a surface.
>
> It is not verified that protons are not quantum particles. If you're saying
> quarks are smaller particles than protons, then give me the mass and charge
> of each right here.

"smaller" or "larger" are concepts difficult to apply at that level.

The currently accepted estimation for quark up is between 1,5 Mev
and 3 Mev and for quark down, between 3 MeV and 9 Mev.

Proton has been verified by non destructive scattering of electrons to
contain only 3 pointlike components which physically deflect electrons
of sufficient energy to penetrate the structure, they were named up
quarks and down quarks: 2 up quarks and 1 down quark. Likewise, neutrons
have been verified to be made up of 1 up quark and 2 down quarks.

The up quark has a confirmed positive charge of 2/3 that of the electron.

The down quark has a confirmed negative charge 1/3 that of the electron.

The charge of the proton is the resultant of the addition of the
fractional charges of the constituting quarks.

2/3 + 2/3 - 1/3 = 3/3 = 1

The charge of the neutron is zero because the fractional charges
cancel each other out.

2/3 - 1/3 - 1/3 = 0

Inelastic back scattering of electrons in those experiments confirms
without a doubt that their mass is in a range close to that of electron,
because masses of 300 MeV or more, as was expected at first would have
resulted in elastic back stattering.

This is public knowledge that you can verify if you really want
to clear this out.

My source for the mass values at this precise moment is
"The European Physical Journal" Volume 15, Number 1-4, 2000, page 382

The rest of the measurable mass of protons and neutrons may well be
caused by the relativistic velocities that up and down quarks possibly
maintain in their dynamic local system. In fact, we don't know yet.

> If the quark theory is correct, and the proton is made
> of 3 of them,

I was not referring to any theory, but to actual scattering experiments.
It is a physically verified fact that protons and neutrons are made up
of 3 sub-particles that can be scattered against by electrons and that
behave, just like electrons, as if they were pointlike, meaning that
they seem not to be themselves made up of smaller particles.

> then at least one of the quarks must have a mass
> greater than 600 times the electron. What is the mass of this particle?

No such particle has been scattered agains inside protons or neutrons.
ergo, it doesn't exist.



> > It is also not really possible for the electron to have a surface.
>
> The charge has an effective surface. It is not a surface of length, but a
> distributed charge. The mass is more like a string.
>
> > However logical your new system may be, if it is not grounded in verified
> > facts about particles, it is bound to remain further away from a true
> > representation of reality than any system that does.
>
> My theory is completely grounded in empirical data. The dimensions are
> exactly as measured and listed at NIST.
>
> If you are not fully instructed in the Quantum Physics Model, but try to
> compare the concepts of the Quantum Physics Model to the concepts of the
> Standard Model, you will only get confused.

I did not try to compare your model with the Standard Model. I just
spotted the points I intervened on in your posts and in the intro of
your text. I did not get confused.

By the way, the Casimir effect is not really a proof of Zero point
energy since it is also perfectly explainable with classical
electrodynamics:

"Electrodynamics of Continuous Media", Landau and Lifshitz, Course of
Theoretical Physics, Volume 8, § 89 et § 90

> There is not a one to one relation between the theories. The Quantum
> Physics Model is a completely different structure for the same exact
> empirical data used for the structure of the Standard Model. The data
> is the same,

Not regarding the known possible mass value range of quarks up and down.

> but the way it is viewed is different. You need to learn
> a completely different paradigm to see the logic of the Quantum Physics
> Model.

I understand that. But before someone will invest the effort to
dig into it, it must at least take into account known non destructive
scattering results on protons.

André Michaud

FrediFizzx

unread,
Jan 2, 2004, 9:38:25 PM1/2/04
to
"Yousuf Khan" <removethisspam.bj...@hotmail.com> wrote in
message
news:vviJb.224907$2We1....@news04.bloor.is.net.cable.rogers.com...

The quarks account for approximately 54 percent of the proton's total
momentum.

FrediFizzx

Y.Porat

unread,
Jan 3, 2004, 12:09:36 PM1/3/04
to
"FrediFizzx" <fredi...@hotmail.com> wrote in message news:<bt59uu$3fh27$1...@ID-185976.news.uni-berlin.de>...
-------------------
the total mass of all quarks is less than 10 percent of Proton mass
yet i suggest that you read
'The third Porat postulation'
and
'the sixth Porat postulate'
is time to end all that mascarad.
---------
all the best
Y.Porat

Y.Porat

unread,
Jan 3, 2004, 12:29:39 PM1/3/04
to
s...@microtec.net (Andr? Michaud) wrote in message news:<562f286c.04010...@posting.google.com>...

> "Dave Thomson" <news3 @ tshankha.no spam.com> wrote in message news:<DnhJb.915$Gn2....@news.uswest.net>...
> > "Andr? Michaud" <s...@microtec.net> wrote in message
> > news
> >
> > It is not verified that protons are not quantum particles. If you're saying
> > quarks are smaller particles than protons, then give me the mass and charge
> > of each right here.
>
> "smaller" or "larger" are concepts difficult to apply at that level.
>
> The currently accepted estimation for quark up is between 1,5 Mev
> and 3 Mev and for quark down, between 3 MeV and 9 Mev.
>
> Proton has been verified by non destructive scattering of electrons to
> contain only 3 pointlike components
------------
realy ?????
have anyone considered the possibility that once you use electron
as 'bullets' *they have no chance* to penertrate deaper
into the protons structure???
as long as not all possible 'bullets have been tried'
you are not alowed to say that it was *proven* that thre are not
more than 3
it might well be that the inner volumes of the proton
do not let any electron to get in there!
and not to forget that we have some 'circomstantial evidence'
that the 3 quarks are not the only entities of the
proton : it was you who taught me that the 3 quarks are
less than 10 percent of the whole mass
(and please dont give me again that ,,, story of relativistic mass
there because it is illogic- no one is weighing permanent
particle mass with some mysterious relativistic entity)
the best thing and the mosrt respossible thing to say as is is:
'the proton inner structure is at least -(if not totally!)-
unknown!

now i would call it, very presumpcious (not you ...)
to have 10 percent in hand' and pretent to claim
any understanding of the whole story


---------
all the best
Y.Porat

------------

Andr? Michaud

unread,
Jan 3, 2004, 11:52:20 PM1/3/04
to
map...@012.net.il (Y.Porat) wrote in message news:<4e35159f.04010...@posting.google.com>...

> s...@microtec.net (Andr? Michaud) wrote in message news:<562f286c.04010...@posting.google.com>...
> > "Dave Thomson" <news3 @ tshankha.no spam.com> wrote in message news:<DnhJb.915$Gn2....@news.uswest.net>...
> > > "Andr? Michaud" <s...@microtec.net> wrote in message
> > > news
> > >
> > > It is not verified that protons are not quantum particles. If you're
> > > saying quarks are smaller particles than protons, then give me the mass
> > > and charge of each right here.
> >
> > "smaller" or "larger" are concepts difficult to apply at that level.
> >
> > The currently accepted estimation for quark up is between 1,5 Mev
> > and 3 Mev and for quark down, between 3 MeV and 9 Mev.
> >
> > Proton has been verified by non destructive scattering of electrons to
> > contain only 3 pointlike components
> ------------
> realy ?????
> have anyone considered the possibility that once you use electron
> as 'bullets' *they have no chance* to penertrate deaper
> into the protons structure???

Quite the opposite. As far as I have been able to ascertain, an electron
with sufficient energy can shoot right trough a proton without hitting
anything if it chances not to back scatter against one of the 3 quarks.

Think that at the energy levels that are required, electrons move at close
to the speed of light.

To my knowledge, the Stanford Linear Accelerator was the first
that could pack enough energy into electrons to have them penetrate
protons.

From what I know, that's how they mapped what was inside protons for the
first time in 1968.

Some electrons were scattered backwards, others were scattered forwards
at various angles which showed the presence of 3 "obstacles".

This is history. You could certainly get independent confirmation of
how they proceeded.

> as long as not all possible 'bullets have been tried'
> you are not alowed to say that it was *proven* that thre are not
> more than 3

Well, protons are out since they are not punctual. As for neutrons, I
don't know how they could control the beam, but they are not punctual
either.

There are not many kinds of "bullets" available for the job, you know.
Positron could also be used and certainly have; and possibly muons,
but those are much less easy to produce.

But whatever punctual bullet they could use, they will always get the
same internal picture of protons. The experiment has been carried out
numerous times, to my knowledge, with always the same result.

There is positively only 3 sub-particles in a protons, 2 up quarks
and 1 down quark.

> it might well be that the inner volumes of the proton


> do not let any electron to get in there!

You can verify this from other sources that you trust. There are
no part of the internal volume of protons that electrons of sufficient
energy cannot shoot through, except of course the location occupied by
the quarks themselves.

> and not to forget that we have some 'circomstantial evidence'
> that the 3 quarks are not the only entities of the
> proton :

Could you be more specific. This is news to me. What circostantial
evidence ?

> it was you who taught me that the 3 quarks are less than 10 percent
> of the whole mass (and please dont give me again that ,,, story of
> relativistic mass there because it is illogic- no one is weighing
> permanent particle mass with some mysterious relativistic entity)

It is very difficult to dispute the real increase in mass of particles
with extreme speeds, because not taking it into account caused the
first circular accelerator at Berkeley to be unable to accelerate
protons at more than even 10 Mev, which by the way is rather low as
energy goes when protons are considered.

It was Bethe who understood that the mass used in calculating the
electrical frequency that should be used to keep them on the optimal
circular trajectory had to be increased by solving the equations
according to the relativistic equations for protons to remain in
phase with the frequency on the optimal circular path.

This was confirmed in 1945 by Veksler and McMillan when they actually
carried out the experiment using this method.

Ever since, particles can be accelerated to the energy limit of
modern circular accelerator and remain in the perfectly circular
path simply because they do take the relativistic increase in mass
into account.

> the best thing and the mosrt respossible thing to say as is is:
> 'the proton inner structure is at least -(if not totally!)-
> unknown!

Absolutely not. As far as the sub-particle contents of protons and
neutrons is concerned, their inner structure is completely understood
since the 1968 confirmation.



> now i would call it, very presumpcious (not you ...)
> to have 10 percent in hand' and pretent to claim
> any understanding of the whole story

No one pretends understanding completely, or even patially for
that matter, why protons and neutrons are made up of only
2 types of quarks: up and down, but the one thing that is
absolutely certain, confirmed many times without the shadow
of a doubt, is that there are only 3 pointlike sub-particles
into a proton or a neutron, and that they have the fractional
charges that I mentioned in my previous post.

As far as orthodox explanatory theories go, to my knowledge,
only one was put forward: Quantum Chromodynamids, and it has
been unsatisfactory from the start and still is.

You could certainly verify all of this with other sources that you
trust.

Regards

André Michaud

FrediFizzx

unread,
Jan 4, 2004, 2:20:19 AM1/4/04
to
"Y.Porat" <map...@012.net.il> wrote in message
news:4e35159f.0401...@posting.google.com...

You are using what is called the "current" masses. The current masses of
quarks are not the "effective" mass when in the proton.

FrediFizzx

FrediFizzx

unread,
Jan 4, 2004, 2:46:48 AM1/4/04
to
"Andr? Michaud" <s...@microtec.net> wrote in message
news:562f286c.0401...@posting.google.com...

Actually Feynman was involved in this and he insisted that they only
consider "partons" when they first did the experiments so as to try to not
bias the experiments in the favor of quarks. I think they actually hit more
than three things in the proton but realized that the portion of momentum
was so low that they were most likely sea-quarks. But all the structure
function math eventually could be explained by three main "partons".

| > as long as not all possible 'bullets have been tried'
| > you are not alowed to say that it was *proven* that thre are not
| > more than 3
|
| Well, protons are out since they are not punctual. As for neutrons, I
| don't know how they could control the beam, but they are not punctual
| either.
|
| There are not many kinds of "bullets" available for the job, you know.
| Positron could also be used and certainly have; and possibly muons,
| but those are much less easy to produce.
|
| But whatever punctual bullet they could use, they will always get the
| same internal picture of protons. The experiment has been carried out
| numerous times, to my knowledge, with always the same result.

They used muon neutrinos to penetrate the proton and this really put the
final cap on the fact that the partons could be explained by quarks.

| There is positively only 3 sub-particles in a protons, 2 up quarks
| and 1 down quark.

Well, there is only 3 main partons. There are some sea-quarks happening
also most likely.

| > it might well be that the inner volumes of the proton
| > do not let any electron to get in there!
|
| You can verify this from other sources that you trust. There are
| no part of the internal volume of protons that electrons of sufficient
| energy cannot shoot through, except of course the location occupied by
| the quarks themselves.

I am not so sure that is exactly true. That is a simplified picture of what
is going on.

| > and not to forget that we have some 'circomstantial evidence'
| > that the 3 quarks are not the only entities of the
| > proton :
|
| Could you be more specific. This is news to me. What circostantial
| evidence ?

Well, it is not even that circumstantial. They do seem to hit sea-quarks in
the scattering experiments.

I don't know if we could really say completely understood, but they
certainly know much more now than ever before.

| > now i would call it, very presumpcious (not you ...)
| > to have 10 percent in hand' and pretent to claim
| > any understanding of the whole story
|
| No one pretends understanding completely, or even patially for
| that matter, why protons and neutrons are made up of only
| 2 types of quarks: up and down, but the one thing that is
| absolutely certain, confirmed many times without the shadow
| of a doubt, is that there are only 3 pointlike sub-particles
| into a proton or a neutron, and that they have the fractional
| charges that I mentioned in my previous post.
|
| As far as orthodox explanatory theories go, to my knowledge,
| only one was put forward: Quantum Chromodynamids, and it has
| been unsatisfactory from the start and still is.

Well, sort of. Like I mentioned earlier, that is why Feynman insisted on
the parton approach. And I think even Gell-mann thought it might be crank
physics when he first came up with it. However, QCD is the best theory we
have so far and explains much about hadronic physics.

FrediFizzx

Y.Porat

unread,
Jan 4, 2004, 8:50:21 AM1/4/04
to
s...@microtec.net (Andr? Michaud) wrote in message news:<562f286c.0401...@posting.google.com>...

> map...@012.net.il (Y.Porat) wrote in message news:
> > > >
> >
> > >
> > > Proton has been verified by non destructive scattering of electrons to
> > > contain only 3 pointlike components
> > ------------
> > realy ?????
> > have anyone considered the possibility that once you use electron
> > as 'bullets' *they have no chance* to penertrate deaper
> > into the protons structure???
>
> Quite the opposite. As far as I have been able to ascertain, an electron
> with sufficient energy can shoot right trough a proton without hitting
> anything if it chances not to back scatter against one of the 3 quarks.
> -----------------
you use what is called a circular logic:
first you asume (an unverified fact) and based on it
you verfy that fact ......
just think:

lets take my approach:
if most of the Proton is unknown:
therfore you cannot predict *what will be the result of a collition
of electrons with that *unknown entity or structure *!!!!
it migh tbe say so small that the electrons say 'pass through it'
without noticing it!...or any other effect that we dont know
because ..... we dont know we have nevrr been ther !(hope you got it)
---------


> Think that at the energy levels that are required, electrons move at close
> to the speed of light.

not relevant using my above logic.
--------


>
> To my knowledge, the Stanford Linear Accelerator was the first
> that could pack enough energy into electrons to have them penetrate
> protons.
>
> From what I know, that's how they mapped what was inside protons for the
> first time in 1968.

-------
they think * they mapped !!
------


>
> Some electrons were scattered backwards, others were scattered forwards
> at various angles which showed the presence of 3 "obstacles".

----
could be because electrons do not interact with the unknown
additinal entity.
----------


>
> This is history. You could certainly get independent confirmation of
> how they proceeded.
>
> > as long as not all possible 'bullets have been tried'
> > you are not alowed to say that it was *proven* that thre are not
> > more than 3
>
> Well, protons are out since they are not punctual. As for neutrons, I
> don't know how they could control the beam, but they are not punctual
> either.

----------
that is only one of the problems.


>
> There are not many kinds of "bullets" available for the job, you know.
> Positron could also be used and certainly have; and possibly muons,
> but those are much less easy to produce.

----------
yes thats part of the problem
there was once a turkish Admiral who was send to conquer Malta
he missed his way came back to his commanders ans declaired:
'malta yok' (Malta is non existant) ........ just a joke
i dont know why i recalled it.
-----------


>
> But whatever punctual bullet they could use, they will always get the
> same internal picture of protons. The experiment has been carried out
> numerous times, to my knowledge, with always the same result.

-----
is it possible to use say gama rays ?/(just a leymans thought)
-------------

>
> There is positively only 3 sub-particles in a protons, 2 up quarks
> and 1 down quark.

---------
have you heared about the finding of a 'penta quark ' lately?
are you sure they are telling you 'anything they find* ?
---------------
------------


>
> > it might well be that the inner volumes of the proton
> > do not let any electron to get in there!
>
> You can verify this from other sources that you trust. There are
> no part of the internal volume of protons that electrons of sufficient
> energy cannot shoot through, except of course the location occupied by
> the quarks themselves.

----------
have you ever tryed to shoot say iron balles using cotton wool bullets...
?
--------


>
> > and not to forget that we have some 'circomstantial evidence'
> > that the 3 quarks are not the only entities of the
> > proton :
>
> Could you be more specific. This is news to me. What circostantial
> evidence ?
>
> > it was you who taught me that the 3 quarks are less than 10 percent
> > of the whole mass (and please dont give me again that ,,, story of
> > relativistic mass there because it is illogic- no one is weighing
> > permanent particle mass with some mysterious relativistic entity)
>
> It is very difficult to dispute the real increase in mass of particles
> with extreme speeds, because not taking it into account caused the
> first circular accelerator at Berkeley to be unable to accelerate
> protons at more than even 10 Mev, which by the way is rather low as
> energy goes when protons are considered.

for me it is very easy to dispute relativistic mass:
1 see 'the third Porst postulation'

now about the circular accelerator i am afraied that
either i expalained it not good enogh
or else you missed my explanation a short time ago:
acording to my claim, that fact that it is more and more difficult
to add acceleration ot a mass, is not mass'inflation'
but more and more *energy 'bullets 'needed because the vleocity
of those bullets is as well limited with the speed of light..
now it is the same with the circular acceleratot;
particles run there first in a straight line than
the accelerator is shooting 'its bullets' *perpendicular*
to their path of movement... the result is change(deviation( in movement
twards the center
now just a methaphore:
suppose a car is running left to right of you.
you have a mission to hit the care with some bullets
the question is :
in which case you will need more bullets to hit it (say per second)
in case one that the car is moving 10 kmh
or in case 2 in which it will move say 300000 kmh??
ps
sory Andre i was born with a nonconformistic mind ......
it is not *your fault* ....... may be mine.....
----------
all the best
Y.Porat
----------------
Regards Yehiel

Y.Porat

unread,
Jan 4, 2004, 8:50:28 AM1/4/04
to
s...@microtec.net (Andr? Michaud) wrote in message news:<562f286c.0401...@posting.google.com>...

> map...@012.net.il (Y.Porat) wrote in message news:
> > > >
> >
> > >
> > > Proton has been verified by non destructive scattering of electrons to
> > > contain only 3 pointlike components
> > ------------
> > realy ?????
> > have anyone considered the possibility that once you use electron
> > as 'bullets' *they have no chance* to penertrate deaper
> > into the protons structure???
>
> Quite the opposite. As far as I have been able to ascertain, an electron
> with sufficient energy can shoot right trough a proton without hitting
> anything if it chances not to back scatter against one of the 3 quarks.
> -----------------
you use what is called a circular logic:
first you asume (an unverified fact) and based on it
you verfy that fact ......
just think:

lets take my approach:
if most of the Proton is unknown:
therfore you cannot predict *what will be the result of a collition
of electrons with that *unknown entity or structure *!!!!
it migh tbe say so small that the electrons say 'pass through it'
without noticing it!...or any other effect that we dont know
because ..... we dont know we have nevrr been ther !(hope you got it)
---------

> Think that at the energy levels that are required, electrons move at close
> to the speed of light.

not relevant using my above logic.
--------
>

> To my knowledge, the Stanford Linear Accelerator was the first
> that could pack enough energy into electrons to have them penetrate
> protons.
>
> From what I know, that's how they mapped what was inside protons for the
> first time in 1968.

-------
they think * they mapped !!
------
>

> Some electrons were scattered backwards, others were scattered forwards
> at various angles which showed the presence of 3 "obstacles".

----
could be because electrons do not interact with the unknown
additinal entity.
----------
>

> This is history. You could certainly get independent confirmation of
> how they proceeded.
>
> > as long as not all possible 'bullets have been tried'
> > you are not alowed to say that it was *proven* that thre are not
> > more than 3
>
> Well, protons are out since they are not punctual. As for neutrons, I
> don't know how they could control the beam, but they are not punctual
> either.

----------
that is only one of the problems.
>

> There are not many kinds of "bullets" available for the job, you know.
> Positron could also be used and certainly have; and possibly muons,
> but those are much less easy to produce.

----------
yes thats part of the problem
there was once a turkish Admiral who was send to conquer Malta
he missed his way came back to his commanders ans declaired:
'malta yok' (Malta is non existant) ........ just a joke
i dont know why i recalled it.
-----------
>

> But whatever punctual bullet they could use, they will always get the
> same internal picture of protons. The experiment has been carried out
> numerous times, to my knowledge, with always the same result.

-----
is it possible to use say gama rays ?/(just a leymans thought)
-------------
>

> There is positively only 3 sub-particles in a protons, 2 up quarks
> and 1 down quark.

---------
have you heared about the finding of a 'penta quark ' lately?
are you sure they are telling you 'anything they find* ?
---------------
------------
>

> > it might well be that the inner volumes of the proton
> > do not let any electron to get in there!
>
> You can verify this from other sources that you trust. There are
> no part of the internal volume of protons that electrons of sufficient
> energy cannot shoot through, except of course the location occupied by
> the quarks themselves.

----------
have you ever tryed to shoot say iron balles using cotton wool bullets...
?
--------
>

> > and not to forget that we have some 'circomstantial evidence'
> > that the 3 quarks are not the only entities of the
> > proton :
>
> Could you be more specific. This is news to me. What circostantial
> evidence ?
>
> > it was you who taught me that the 3 quarks are less than 10 percent
> > of the whole mass (and please dont give me again that ,,, story of
> > relativistic mass there because it is illogic- no one is weighing
> > permanent particle mass with some mysterious relativistic entity)
>
> It is very difficult to dispute the real increase in mass of particles
> with extreme speeds, because not taking it into account caused the
> first circular accelerator at Berkeley to be unable to accelerate
> protons at more than even 10 Mev, which by the way is rather low as
> energy goes when protons are considered.

Andr? Michaud

unread,
Jan 4, 2004, 11:33:19 AM1/4/04
to
"FrediFizzx" <fredi...@hotmail.com> wrote in message news:<bt8gd5$4c55v$1...@ID-185976.news.uni-berlin.de>...

Careful to leave some work for future generations, I guess :-]

> I think they actually hit more than three things in the proton but
> realized that the portion of momentum was so low that they were most
> likely sea-quarks. But all the structure function math eventually
> could be explained by three main "partons".

But don't you think that if something else than the up and down quarks
were part of the proton structure, it would eventually have stood out
with some reasonable degree of consistency in the numerous scattering
experiments that were carried out over the extended 30+ years period
since the first SLAC experiment?

I never came across any paper that even suggested such consistency
other than that regarding the "three main partons", as you say.

I personally tend to take into account only consistent scattering
results.

> | > as long as not all possible 'bullets have been tried'
> | > you are not alowed to say that it was *proven* that thre are not
> | > more than 3
> |
> | Well, protons are out since they are not punctual. As for neutrons, I
> | don't know how they could control the beam, but they are not punctual
> | either.
> |
> | There are not many kinds of "bullets" available for the job, you know.
> | Positron could also be used and certainly have; and possibly muons,
> | but those are much less easy to produce.
> |
> | But whatever punctual bullet they could use, they will always get the
> | same internal picture of protons. The experiment has been carried out
> | numerous times, to my knowledge, with always the same result.
>
> They used muon neutrinos to penetrate the proton and this really put the
> final cap on the fact that the partons could be explained by quarks.
>
> | There is positively only 3 sub-particles in a protons, 2 up quarks
> | and 1 down quark.
>
> Well, there is only 3 main partons. There are some sea-quarks happening
> also most likely.

Ref: my question above.



> | > it might well be that the inner volumes of the proton
> | > do not let any electron to get in there!
> |
> | You can verify this from other sources that you trust. There are
> | no part of the internal volume of protons that electrons of sufficient
> | energy cannot shoot through, except of course the location occupied by
> | the quarks themselves.
>
> I am not so sure that is exactly true. That is a simplified picture of
> what is going on.
>
> | > and not to forget that we have some 'circomstantial evidence'
> | > that the 3 quarks are not the only entities of the
> | > proton :
> |
> | Could you be more specific. This is news to me. What circostantial
> | evidence ?
>
> Well, it is not even that circumstantial. They do seem to hit sea-quarks in
> the scattering experiments.

I am intrigued. Still relating to my question above, would you have
any reference I could read that would reveal some consistency of
"anomalous" scattering results over the course of the past 30 years?

Interesting! I knew of Feynman's reservation, but I was not aware of
Gell-man's opinion on the matter. I am positive that Zweig would also
have wanted the question completely cleared out. I personally would
have liked them to go all the way also.

> However, QCD is the best theory we have so far and explains much about
> hadronic physics.

Much, but unfortunately not all, even after 30 years, despite the
totally adequate experimental means that we have had at our disposal
over that period.

We need some new approach, possibly going back to fundamentals.

For example maybe doing away with all virtual particles and try
to deal again directly with the actual forces that they are meant
to represent in equations.

Barring some paper that would confirm the consistent presence of
something specific other than the three confirmed quarks, my personal
view is likely to remain that a proton is simply those 3 quarks anchored
by resonance to a fundamental least action level in a local dynamic
system, unable to escape and unable to get closer to each other,
except to fleetingly occupy a limited range of metastable states
when excited by inelastic collisions or gammas.

André Michaud

Giovanni

unread,
Jan 4, 2004, 7:10:55 PM1/4/04
to
> > >
> > > The currently accepted estimation for quark up is between 1,5 Mev
> > > and 3 Mev and for quark down, between 3 MeV and 9 Mev.
> > >
> > > Proton has been verified by non destructive scattering of electrons to
> > > contain only 3 pointlike components
> > ------------
> > realy ?????
> > have anyone considered the possibility that once you use electron
> > as 'bullets' *they have no chance* to penertrate deaper
> > into the protons structure???
>
> Quite the opposite. As far as I have been able to ascertain, an electron
> with sufficient energy can shoot right trough a proton without hitting
> anything if it chances not to back scatter against one of the 3 quarks.
>
> Think that at the energy levels that are required, electrons move at close
> to the speed of light.
>
> To my knowledge, the Stanford Linear Accelerator was the first
> that could pack enough energy into electrons to have them penetrate
> protons.
>
> From what I know, that's how they mapped what was inside protons for the
> first time in 1968.

The quark discovery is like a bullet hitting one of the set of
billiard balls instead of using a cue ball. Unfortunately, the bullet
could only "fission" one of the balls and the rest of the balls have
not been traced and called it sea quark?

That was for proton. For neutron and still using the same
bullet(electron), the electron always hit the proton in the neutron
due to their attraction. The up quarks however became the down quarks
and the down quark became the up quark. Why? Simply because the proton
in the neutron is in inverted position hence the same particles but
different scatttering direction.

Gell-Man and Zweig thought that the structure in elementary particles
could be likened to the structure of atoms. But they neglect that
atomic particles are distinct and the third is a combination of the
two. They even didn't consider that only two distinct particles is the
least combination there is and that more than that is the addition of
the combination of the two and not the addition of the same of either
particle.

>
> Some electrons were scattered backwards, others were scattered forwards
> at various angles which showed the presence of 3 "obstacles".

The energy of the electron could only break up one of the clusters of
particles in a hadron leaving the others undetected and have been
called other names to it.

>
> But whatever punctual bullet they could use, they will always get the
> same internal picture of protons. The experiment has been carried out
> numerous times, to my knowledge, with always the same result.

The same mistake.


>
> There is positively only 3 sub-particles in a protons, 2 up quarks
> and 1 down quark.

As I said in neutron the 2 down quarks and 1 up quark is the same
particles in proton which are the 2 up quarks and 1 down quark
respectively. The difference is their scattering directions and the
arbitrary assignment of charges.

>
> You can verify this from other sources that you trust. There are
> no part of the internal volume of protons that electrons of sufficient
> energy cannot shoot through, except of course the location occupied by
> the quarks themselves.

>

> Absolutely not. As far as the sub-particle contents of protons and
> neutrons is concerned, their inner structure is completely understood
> since the 1968 confirmation.

Not accurate.


>
>
> No one pretends understanding completely, or even patially for
> that matter, why protons and neutrons are made up of only
> 2 types of quarks: up and down, but the one thing that is
> absolutely certain, confirmed many times without the shadow
> of a doubt, is that there are only 3 pointlike sub-particles
> into a proton or a neutron, and that they have the fractional
> charges that I mentioned in my previous post.

Why would a high energy particle hit only the small ones and not hit
the bigger masses? It is harder to play with duckpins than ten pins.

FrediFizzx

unread,
Jan 4, 2004, 9:53:47 PM1/4/04
to
"Andr? Michaud" <s...@microtec.net> wrote in message
news:562f286c.04010...@posting.google.com...

See below.

Any good book on particle physics should have some basic scattering data in
it. Mind you, this is mostly an effect of the violations of scaling. As
the probe has higher and higher energy, they can see more details of the
internal processes of a nucleon. There is a lot more going on in a nucleon
than just three quarks bouncing around. A gluon can briefly become a quark
pair and this does in fact get hit with higher energy probes.

From what I can tell, they are doing all kinds of approaches now-a-days.
Maybe you just need to research some more on what is actually going on out
there. All the different kind of experiments is mind boggling. Every time
I turn around I discover some different kind of experiment going on.

| For example maybe doing away with all virtual particles and try
| to deal again directly with the actual forces that they are meant
| to represent in equations.

Well, to me, virtual particles are a representation of the quantum vacuum so
I don't think it would be wise to not consider virtual particles.

| Barring some paper that would confirm the consistent presence of
| something specific other than the three confirmed quarks, my personal
| view is likely to remain that a proton is simply those 3 quarks anchored
| by resonance to a fundamental least action level in a local dynamic
| system, unable to escape and unable to get closer to each other,
| except to fleetingly occupy a limited range of metastable states
| when excited by inelastic collisions or gammas.

Well, we just can't ignore the QCD vacuum and pretend it doesn't exist. You
either have to have virtual particles or you have to have a vacuum medium.
Otherwise, we would have to resort to magic to explain what is going on. My
advice is to realize that charge of any form has to be a mechanical action
of some type. There is no magic.

FrediFizzx

Andr? Michaud

unread,
Jan 5, 2004, 1:38:19 AM1/5/04
to
"FrediFizzx" <fredi...@hotmail.com> wrote in message news:<btajjm$566ok$1...@ID-185976.news.uni-berlin.de>...

> "Andr? Michaud" <s...@microtec.net> wrote in message
> news:562f286c.04010...@posting.google.com...

[Major snippage. This thing was getting huge!]

> | I am intrigued. Still relating to my question above, would you have
> | any reference I could read that would reveal some consistency of
> | "anomalous" scattering results over the course of the past 30 years?
>
> Any good book on particle physics should have some basic scattering data in
> it. Mind you, this is mostly an effect of the violations of scaling. As
> the probe has higher and higher energy, they can see more details of the
> internal processes of a nucleon. There is a lot more going on in a nucleon
> than just three quarks bouncing around. A gluon can briefly become a quark
> pair and this does in fact get hit with higher energy probes.

I am familiar. We know however that any time sufficient energy is
locally concentrated, all particle combinations whose mass requires
less energy than that available are likely to spontaneously appear.

The more energy is concentrated at some point in space, the more
parasitic particles fleetingly occur.

I had a very enlightening discussion regarding this some years ago
with a physicist from DESY.

My view is that the only really significant exploratory scatterings are
those that feed in as little excess energy as possible. Excess energy
not being sufficient then to convert to parasitic unwanted particles
that would not have been there to start with.

This seems to be the only safe way to ascertain the real contents of
complex particles.

[snip]

> | We need some new approach, possibly going back to fundamentals.
>
> From what I can tell, they are doing all kinds of approaches now-a-days.
> Maybe you just need to research some more on what is actually going on out
> there. All the different kind of experiments is mind boggling. Every time
> I turn around I discover some different kind of experiment going on.

Yes, plenty of action, which I keep an eye on.



> | For example maybe doing away with all virtual particles and try
> | to deal again directly with the actual forces that they are meant
> | to represent in equations.
>
> Well, to me, virtual particles are a representation of the quantum
> vacuum so I don't think it would be wise to not consider virtual
> particles.

I see. So you berth at the QFT wharf.

Quite respectable.

Not my piece of cake though.

> | Barring some paper that would confirm the consistent presence of
> | something specific other than the three confirmed quarks, my personal
> | view is likely to remain that a proton is simply those 3 quarks anchored
> | by resonance to a fundamental least action level in a local dynamic
> | system, unable to escape and unable to get closer to each other,
> | except to fleetingly occupy a limited range of metastable states
> | when excited by inelastic collisions or gammas.
>
> Well, we just can't ignore the QCD vacuum and pretend it doesn't exist.
> You either have to have virtual particles or you have to have a vacuum
> medium.

I wouldn't bet on the idea that these two are the only possible
alternatives.

Even Feynman was careful in his 1949 paper to clearly state that his
virtual photons were a metaphor for the underlying Coulomb interaction,
which, to my knowledge does not require a vacuum medium, or rather,
can be perceived as acting across totally empty vacuum.

I have a view that the general acceptance of the static Lagrangian method
instead of the dynamic Hamiltonian method as suggested by Feynman in the
framework of his definition of QED, has been instrumental in a general
loss of interest for the fact that scattering and collisions between
particles are precise temporal sequences of events.

He went so far as to directly state it in his 49 paper:

"In many problems, for example, the close collisions of particles, we
are not interested in the precise temporal sequence of events. It is
of no interest to be able to say how the situation would look at each
instant of time during a collision and how it progresses from instant
to instant."

"Space-Time Approach to Quantum Electrodynamics", Richard Feynman,
Phys. Rev. 76, 769 (1949), page 771

I happen to disagree with him on this point.

I even think that this research philosophy, which apparently took roots,
has induced the following generations of physicists to refrain from
exploring the only remaining unexplored frontier in fundamental physics
for the past 50 years.

> Otherwise, we would have to resort to magic to explain what is going on.
> My advice is to realize that charge of any form has to be a mechanical
> action of some type. There is no magic.

I fully agree that no magic can be involved.

I happen to be a causalist.

André Michaud

Y.Porat

unread,
Jan 5, 2004, 2:10:52 AM1/5/04
to
giovanni_...@yahoo.it (Giovanni) wrote in message news:<d57f2d68.04010...@posting.google.com>...

> > > >
> > > > The currently accepted estimation for quark up is between 1,5 Mev
> > > > and 3 Mev and for quark down, between 3 MeV and 9 Mev.
> > > >
>
>
>
>
>
>
>
>
> > absolutely certain, confirmed many times without the shadow
> > of a doubt, is that there are only 3 pointlike sub-particles
> > into a proton or a neutron, and that they have the fractional
> > charges that I mentioned in my previous post.
>
> Why would a high energy particle hit only the small ones and not hit
> the bigger masses? It is harder to play with duckpins than ten pins.
-------------------
yes !
i think you got it :
the moment it was found that the quark mass is only less than
10 percent of the nucleid mass, the quark model was doomed!
sory it takes so long for people to get it.
if i have to reexplain it i will do it shortly:

1 the quarks mass is less than 10 percent of the nucleid
2 that means *90 percent is missing*

3 now the 'smart guys' tell you - we have gluons (??????)
let it be 'gluons' but gluons is exactly equivalent to - *unknown*

4 some other smary guys tell you: it is 'relativistic mass'?
ie quarkd due to their motion 'enlarge their mass'
but
no one ever while dwaling with any mass evaluation 'of a particle'
*is taking in acount the inner motion in it*
while you take a nuclei you dont say the mass of the proton is bigger
than 1 amu 'bacause ther is an inner motion in it '
in short 'inner motion' is not taken in acount in mass evaluation.

4 acording to the 'the third Porat postulation'
*there is no 'relativistic mass' anywhere and whatsoever!
it was justs misinterpretation of the E=Lf MC^2 equation
so ....
as you say we have in hand 10 percent and another 90 percent
missing !!!
that is what is called 'knowing the nucleid' ???
what is it cheating ??? stupidity ???
who knowns?
-------------------------

FrediFizzx

unread,
Jan 5, 2004, 3:03:33 AM1/5/04
to

Yes I agree, I think more lower energy experiments are needed. A tricky
business but we should have some really clever people out there to figure
out ways of doing them now-a-days.

| [snip]
|
| > | We need some new approach, possibly going back to fundamentals.
| >
| > From what I can tell, they are doing all kinds of approaches now-a-days.
| > Maybe you just need to research some more on what is actually going on
out
| > there. All the different kind of experiments is mind boggling. Every
time
| > I turn around I discover some different kind of experiment going on.
|
| Yes, plenty of action, which I keep an eye on.
|
| > | For example maybe doing away with all virtual particles and try
| > | to deal again directly with the actual forces that they are meant
| > | to represent in equations.
| >
| > Well, to me, virtual particles are a representation of the quantum
| > vacuum so I don't think it would be wise to not consider virtual
| > particles.
|
| I see. So you berth at the QFT wharf.

Not entirely.

| Quite respectable.
|
| Not my piece of cake though.

I am trying to learn about many different ways of looking at physics and
extracting the best ideas.

| > | Barring some paper that would confirm the consistent presence of
| > | something specific other than the three confirmed quarks, my personal
| > | view is likely to remain that a proton is simply those 3 quarks
anchored
| > | by resonance to a fundamental least action level in a local dynamic
| > | system, unable to escape and unable to get closer to each other,
| > | except to fleetingly occupy a limited range of metastable states
| > | when excited by inelastic collisions or gammas.
| >
| > Well, we just can't ignore the QCD vacuum and pretend it doesn't exist.
| > You either have to have virtual particles or you have to have a vacuum
| > medium.
|
| I wouldn't bet on the idea that these two are the only possible
| alternatives.
|
| Even Feynman was careful in his 1949 paper to clearly state that his
| virtual photons were a metaphor for the underlying Coulomb interaction,
| which, to my knowledge does not require a vacuum medium, or rather,
| can be perceived as acting across totally empty vacuum.

What other ways could there be? You either have to have a medium or you
have to have particles colliding with each other for action across a
distance. Somehow an electron "knows" when an other electron is around.

| I have a view that the general acceptance of the static Lagrangian method
| instead of the dynamic Hamiltonian method as suggested by Feynman in the
| framework of his definition of QED, has been instrumental in a general
| loss of interest for the fact that scattering and collisions between
| particles are precise temporal sequences of events.
|
| He went so far as to directly state it in his 49 paper:
|
| "In many problems, for example, the close collisions of particles, we
| are not interested in the precise temporal sequence of events. It is
| of no interest to be able to say how the situation would look at each
| instant of time during a collision and how it progresses from instant
| to instant."
|
| "Space-Time Approach to Quantum Electrodynamics", Richard Feynman,
| Phys. Rev. 76, 769 (1949), page 771
|
| I happen to disagree with him on this point.
|
| I even think that this research philosophy, which apparently took roots,
| has induced the following generations of physicists to refrain from
| exploring the only remaining unexplored frontier in fundamental physics
| for the past 50 years.

Kind of ironic because that is probably the last thing Feynman would have
wanted that I get from reading his nobel lecture.

| > Otherwise, we would have to resort to magic to explain what is going on.
| > My advice is to realize that charge of any form has to be a mechanical
| > action of some type. There is no magic.
|
| I fully agree that no magic can be involved.
|
| I happen to be a causalist.

I think I am still sitting on the fence with the issue of cause. Maybe
leaning towards quantum objects can just decay randomly. It has to do with
my idea that time doesn't exist for a single quantum object. So if we have
a strange mixture of time and no time at the quantum level, then I suppose
random events could happen. But then it doesn't fit well with my idea of an
organized quantum vacuum either. Well, maybe it does since all my motion in
this idea is massless "particles" moving at c.

FrediFizzx

Andr? Michaud

unread,
Jan 5, 2004, 12:32:55 PM1/5/04
to
map...@012.net.il (Y.Porat) wrote in message news:<4e35159f.04010...@posting.google.com>...

But, my friend, the inertia caused by what you call 'inner motion'
in the local confines of a particle IS the mass that we measure.

Regards

André Michaud

Andr? Michaud

unread,
Jan 5, 2004, 1:33:05 PM1/5/04
to
"FrediFizzx" <fredi...@hotmail.com> wrote in message news:<btb5om$56fhb$1...@ID-185976.news.uni-berlin.de>...

> "Andr? Michaud" <s...@microtec.net> wrote in message
> news:562f286c.04010...@posting.google.com...
> | "FrediFizzx" <fredi...@hotmail.com> wrote in message
> news:<btajjm$566ok$1...@ID-185976.news.uni-berlin.de>...
> | > "Andr? Michaud" <s...@microtec.net> wrote in message
> | > news:562f286c.04010...@posting.google.com...
> |
> | [Major snippage. This thing was getting huge!]
> |
> |
> | The more energy is concentrated at some point in space, the more
> | parasitic particles fleetingly occur.
> |
> | I had a very enlightening discussion regarding this some years ago
> | with a physicist from DESY.
> |
> | My view is that the only really significant exploratory scatterings are
> | those that feed in as little excess energy as possible. Excess energy
> | not being sufficient then to convert to parasitic unwanted particles
> | that would not have been there to start with.
> |
> | This seems to be the only safe way to ascertain the real contents of
> | complex particles.
>
> Yes I agree, I think more lower energy experiments are needed. A tricky
> business but we should have some really clever people out there to figure
> out ways of doing them now-a-days.

Absolutely. When the number of experimentalists who share this opinion
become sufficient to influence the work schedule, progress will definitely
be made in this area.



> | [snip]
> |
> | > | We need some new approach, possibly going back to fundamentals.
> | >
> | > From what I can tell, they are doing all kinds of approaches now-a-days.
> | > Maybe you just need to research some more on what is actually going on
> | > out there. All the different kind of experiments is mind boggling.
> | > Every time I turn around I discover some different kind of experiment
> | > going on.
> |
> | Yes, plenty of action, which I keep an eye on.
> |
> | > | For example maybe doing away with all virtual particles and try
> | > | to deal again directly with the actual forces that they are meant
> | > | to represent in equations.
> | >
> | > Well, to me, virtual particles are a representation of the quantum
> | > vacuum so I don't think it would be wise to not consider virtual
> | > particles.
> |
> | I see. So you berth at the QFT wharf.
>
> Not entirely.
>
> | Quite respectable.
> |
> | Not my piece of cake though.
>
> I am trying to learn about many different ways of looking at physics and
> extracting the best ideas.

Definitely the way to go.

[snip]



> |
> | Even Feynman was careful in his 1949 paper to clearly state that his
> | virtual photons were a metaphor for the underlying Coulomb interaction,
> | which, to my knowledge does not require a vacuum medium, or rather,
> | can be perceived as acting across totally empty vacuum.
>
> What other ways could there be? You either have to have a medium or you
> have to have particles colliding with each other for action across a
> distance.

Well, these two options obviously allow extensive coverage of the problems
at hand, and actually served as a foundation for all of our current
theories.

There is no question also that all the maths that was developped and do
work will always be useful.

You will notice however that both of these assume that particles are
singularities in an underlying field (Maxwell's electromagnetic field).
For the past century, physicist have associated the electromagnetic
characteristics of energy, not directly to the particles, but to the
underlying field, which is considered more fundamental.

As I toured all of the pertinent available literature, I noticed that
one major (in my view) avenue has never been seriously considered at
the foundamental level. I could locate no trace of any analysis in
this regard.

What if there actually existed no such underlying field ?

Particles would suddenly cease to be singularities in a suddenly
non-existent underlying wave phenomenon, to become full fledged
electromagnetic entities, which would make the whole set of these
particles and their interactions, the only possible foundation for
a revamped Maxwellian electromagnetic structure.

Such a direction involves however eventually reconsidering space
geometry at the foundamental level, which definitely is the ultimate
mindbender, but which can lead to surprisingly interesting results.

But I know of no one who had become comfortable with Minkowsky's
space geometry and QED to ever consider risk losing footing again
by rethinking the whole sheebang from the ground up.

Unfortunately, orthodox theories, despite the remaining problems, always
seem too satisfactory, and life too short, for the required time and
effort to be invested.

> Somehow an electron "knows" when an other electron is around.

I would formulate this as "they both reciprocally interact".

> | I have a view that the general acceptance of the static Lagrangian method
> | instead of the dynamic Hamiltonian method as suggested by Feynman in the
> | framework of his definition of QED, has been instrumental in a general
> | loss of interest for the fact that scattering and collisions between
> | particles are precise temporal sequences of events.
> |
> | He went so far as to directly state it in his 49 paper:
> |
> | "In many problems, for example, the close collisions of particles, we
> | are not interested in the precise temporal sequence of events. It is
> | of no interest to be able to say how the situation would look at each
> | instant of time during a collision and how it progresses from instant
> | to instant."
> |
> | "Space-Time Approach to Quantum Electrodynamics", Richard Feynman,
> | Phys. Rev. 76, 769 (1949), page 771
> |
> | I happen to disagree with him on this point.
> |
> | I even think that this research philosophy, which apparently took roots,
> | has induced the following generations of physicists to refrain from
> | exploring the only remaining unexplored frontier in fundamental physics
> | for the past 50 years.
>
> Kind of ironic because that is probably the last thing Feynman would have
> wanted that I get from reading his nobel lecture.

Indeed. But Heisenberg's uncertainty principle, taken as a physical
limitation rather than a limitation of the method definitely also
played a role.



> | > Otherwise, we would have to resort to magic to explain what is going on.
> | > My advice is to realize that charge of any form has to be a mechanical
> | > action of some type. There is no magic.
> |
> | I fully agree that no magic can be involved.
> |
> | I happen to be a causalist.
>
> I think I am still sitting on the fence with the issue of cause. Maybe
> leaning towards quantum objects can just decay randomly. It has to do
> with my idea that time doesn't exist for a single quantum object. So if
> we have a strange mixture of time and no time at the quantum level, then
> I suppose random events could happen. But then it doesn't fit well with
> my idea of an organized quantum vacuum either. Well, maybe it does since
> all my motion in this idea is massless "particles" moving at c.

Massless definitely points to moving at c, no question.

I also was quite fascinated for a time by Stükelberg's idea that
positrons were electrons moving backwards in time.

In fact, one can build any number of quite logical and satisfactory models
on premises that we define.

There seems to exist however only one external reality, which we are
part of, and that I think we stand to gain immensely by identifying
properly the real set of fundamental premises.

I think we have enough data now to really give a good shove to the job.

André Michaud

Y.Porat

unread,
Jan 6, 2004, 11:01:04 AM1/6/04
to
s...@microtec.net (Andr? Michaud) wrote in message news:<562f286c.0401...@posting.google.com>...
---------------------
1 see 'The third Porat Postulation'

there is no 'relativistic mass' a shoking idea? once it will be
well undestood, suddenly many 'fathers' of that idea will pop up
and say of course, it is so obvious ........ i saied it long ago ....

2
lets take for instace the electron mass, it is always detected
in movement right ?
now anyone ever took the movement of the electron, in its mass
calculation?
or another way:
while we evaluate the mass of an atom- do we take in acount
the movement of the electrons ?

3 dont forget that even the existing explanation
of the nucleid mass 'fill it ' with 'Gluons'
what are those gluons??!!
is it enough just to invent a name and throw it in the air?
or elase we whant experimental evidence for it
as long as there is no prove of it - it is just mumbling isnt that?

4 pleae dont forget the *proportions*:
10 percent 'known' 90 percent *unknown*!
(or if you want at the good case 90 percent speculative )
is that scince ???

just immagine that not famous people would suggest such a thing
but say .. me and you .. what would be the way such a theory
would be accepted
(i am sure we would be stoned immediately .....)
sort of , the more presumptious you are , the more 'credible'
you are accepted.
--------------------------
Regards
Y.porat
---------------

Michael Moroney

unread,
Jan 6, 2004, 3:25:13 PM1/6/04
to
map...@012.net.il (Y.Porat) writes:

>> But, my friend, the inertia caused by what you call 'inner motion'
>> in the local confines of a particle IS the mass that we measure.

>lets take for instace the electron mass, it is always detected


>in movement right ?
>now anyone ever took the movement of the electron, in its mass
>calculation?
>or another way:
>while we evaluate the mass of an atom- do we take in acount
>the movement of the electrons ?

First electron binding to an atom is a few eV, if you calculate the mass
of that energy it's very small.

Second, if you want high precision you _would_ take into account the
motion energy.

Consider also particle accelerators. At low energies, you can just plug
in the electron mass into the calculations to compute the voltages/
magnetic field strengths etc. to get the thing to work (such as an
electron gun in a TV). At very high energies that fails completely. The
electrons are moving at close to c, and are more massive than an at-rest
electron. Very much more for very high energies, so much so that the rest
mass of an electron is a tiny percentage of its total mass!
An electron with 10% of its mass its rest mass and the other 90% due to
its kinetic energy is _easily_ produced. It even happens naturally in
some beta decaying isotopes (5 MeV or more decay energy)
--
-Mike

Andr? Michaud

unread,
Jan 6, 2004, 6:46:40 PM1/6/04
to

Yes. There is no other way. But it doesn't mean that it is not
very precise.

> now anyone ever took the movement of the electron, in its mass
> calculation?

Well, for low velocities, that energy is negligible and the
measured mass just about equates rest mass.

In 1910 (if I remember correctly) Millikan calculated very
precisely from experiments the charge of an electron.

With this knowledge, it was possible to calculate with the
highest precision the "rest mass" of electrons in Wilson chambers

Here is a link to a little drawing I scanned fo you to illustrate
the general setup inside a Wilson chamber.

http://pages.globetrotter.net/srp/chwilson.htm

All electrons having the exact same charge and exact same mass
at low energies, their velocity and the strength of the electromagnet
field could be set with high precision, the only unknown being their
mass, the length of their trajectories allowed very precise calculation
of said masses.

When they are accelerated to higher velocities, part of that energy
combines with what you call their "inner energy" and so contributes
to increase their real mass. That excess mass is lost as soon as the
electron is slowed down, but it can never go below .511 MeV which
would be the electron energy if it were to completely stop moving.

André Michaud

Y.Porat

unread,
Jan 7, 2004, 4:38:30 AM1/7/04
to
s...@microtec.net (Andr? Michaud) wrote in message news:<562f286c.04010...@posting.google.com>...

> map...@012.net.il (Y.Porat) wrote in message news:<4e35159f.04010...@posting.google.com>...
> > s...@microtec.net (Andr? Michaud) wrote in message news:<562f286c.0401...@posting.google.com>...
> >
> > > > > Why would a high energy particle hit only the small ones and not hit
> > > > > the bigger masses? It is harder to play with duckpins than ten pins.
> > > > -------------------
> > > > yes !
> > > > i think you got it :
> > > > the moment it was found that the quark mass is only less than
> > > > 10 percent of the nucleid mass, the quark model was doomed!
> > > > sory it takes so long for people to get it.
> > > > if i have to reexplain it i will do it shortly:
> > > >
> > > > 1 the quarks mass is less than 10 percent of the nucleid
> > > > 2 that means *90 percent is missing*
> > > >
> > > > 3 now the 'smart guys' tell you - we have gluons (??????)
> > > > let it be 'gluons' but gluons is exactly equivalent to - *unknown*
> > > >
> > > > 4 some other smary guys tell you: it is 'relativistic mass'?
> > > > ie quarkd due to their motion 'enlarge their mass'
> > > > but
> > > > no one ever while dwaling with any mass evaluation 'of a particle'
> > > > *is taking in acount the inner motion in it*
> > > > while you take a nuclei you dont say the mass of the proton is bigger
> > > > than 1 amu 'bacause ther is an inner motion in it '
> > > > in short 'inner motion' is not taken in acount in mass evaluation.
> > >
> >
> > 1 see 'The third Porat Postulation'
> >
> > there is no 'relativistic mass' a shoking idea? once it will be
> > well undestood, suddenly many 'fathers' of that idea will pop up
> > and say of course, it is so obvious ........ i saied it long ago ....
> >
> >
> Yes. There is no other way. But it doesn't mean that it is not
> very precise.
>
> > now anyone ever took the movement of the electron, in its mass
> > calculation?
>
> Well, for low velocities, that energy is negligible and the
> measured mass just about equates rest mass.
>
>
> Here is a link to a little drawing I scanned fo you to illustrate
> the general setup inside a Wilson chamber.
>
> http://pages.globetrotter.net/srp/chwilson.htm
>
> All electrons having the exact same charge and exact same mass
> at low energies, their velocity and the strength of the electromagnet
> field could be set with high precision, the only unknown being their
> mass, the length of their trajectories allowed very precise calculation
> of said masses.
>
> When they are accelerated to higher velocities, part of that energy
> combines with what you call their "inner energy" and so contributes
> to increase their real mass. That excess mass is lost as soon as the
> electron is slowed down, but it can never go below .511 MeV which
> would be the electron energy if it were to completely stop moving.
>
> André Michaud
-------------
i would say at first glance that your logic works against you.:
the more general equation of energy is :
(E^2)times LF = (mc^)^2 + (pc )^2
in words
it is a vector combination of two vectors *added perpendicularly*
ie the momentum vector is independant of the rest mass vector
(an addition of momentum doea not add to the rest mass
not to mentin that in my formula Lf is only at the 'right side of
the formula) nothing of it in the left side.
if energy is added, it is added only to the momentum part of it
now no ne agrues that the more mo0mentum you add
the trijectory of the particle will be more 'flat'
now why do i say it works againt your logic?:
while you detect a quark, is it with its orriginal energy
or with additional energy ? - obviously with additional
energy
now what is its original situation in the nucleid?
is it with the additional energy that was used during its 'discovery'?
no it is without that addition
so will it be in the nuleid more massive or less masive- acording
to *your concept*/
in the nucleid it should be less massive
now
in the more massive situation it is only a few percent of the nucleid mass
so what will be in its less masive situation? *even less than
those few miserable percent*!

now lets examine anothe rexample:
lets take an Ione of an element
and measure its mass
now lets take the same element with say an Ione that is missing
say 3 electrons
now the questuion is : what will be the mass diffwerence
between case no one
and case no 2 with the 3 electrons missing
will it be a difference of 3 electron mass
or will it be a difference of 3 electrons *+* some addition
of any relativistic mass (ie more that 3 electrons mass) ???
i think the answer is clear - it is experimental facts.
all the best
Y.Porat
---------

Y.Porat

unread,
Jan 7, 2004, 5:40:49 AM1/7/04
to
mor...@world.std.spaamtrap.com (Michael Moroney) wrote in message news:<btf5j9$28c$2...@pcls4.std.com>...
----------------
Hi Mike
as i understand from Bjoern you got a 'copy of copy' of my book
i hope a complete one wonder how ledgible it is that copy of copy.

2 see my explanations to Andre
for instance while we evaluate the mass of an atom
we do not consider binding energies or alike we take it
as is without lo0oking inside

once you have an ione with one electron less than the 'ordinary'
the differenc inmass will be just 'pure flat electron mass...'
nothing else ie we measure it in its (not sure how it is called)
its say rest position no additional energy is attached to it
the rest mass energy and the additional energy
are *perpendicular vectors ie they are independant of each other
so once you add energy to a particle you add it to increase velocity
(no connection to the 'inner velocity' that makes mc^2
iotis added only to the say out velocity- ie perpendiculas vectors.
in which onlt the perpendicular additional vector is increeses
that changes nothing inside the particle or in its components
they stay constant, (probably with an intrinsic velocity of light
*that cannot be augmented* IOW you cannot add anything more
on thatmaximum intrinsic velocity movement.(probably circular)
you can accelerate that circular inner moving subparticle
only its center of motion
so you have a combined motion:
one at the circle (unchanged) and an additional motion
of the particle linearily (the center moves linearily
say methaphorically the earth:
rotating around itself and in addition doing an orbit
around the sun,
all that last explanation is of course my private vision of it
not in detales obligatiry proven, just to illustrate
my more abstract ideas.about what is realy rest mass
and 'relativistic mass' i think i proved it in my third postulation
i will have to expand it to the more general case
in which the pc term is added.
the additional energy is only to the pVc' part and not even to
all the ;pc' but only to the V part of it ie never to the m part of it

Andr? Michaud

unread,
Jan 7, 2004, 1:07:20 PM1/7/04
to

Absolute agreement.

> (an addition of momentum doea not add to the rest mass

If real energy is involved, there is no way for it not to
seek internal electromagnetic equilibrium. This means that
part of the added energy will be expressed as momentum and
part of it will quantize orthogonally to maintain this
equilibrium. That second part has no way other than to
associate through antiparallel spin with the already
orthogonally quantized energy that constitutes what we call
the rest mass of the particle.

That part of the added energy that quantizes so that equilibrium
is maintained becomes measurable as added mass to the rest mass
of the particle.

> not to mentin that in my formula Lf is only at the 'right side of
> the formula) nothing of it in the left side.

I do not know what your LF means, but the original formula empirically
takes into account the automatic quantization of part of the added
energy, at the general level. This is why it gives exacts results when
velocities are calculated for any amount of added energy.

But I personally prefer the following form, which allows directly
calculating velocities and is very easy to use:

v = c(sqrt(1-(mc^2/(K+mc^2))^2)

For example, if you induce 10 MeV of energy into an electron, you get

v = c(sqrt(1-(0,511/(10+0,511))^2)

v= c(,998817556)

v=299437972 m/s very close to the speed of light

It is easy to substitute any energy value

> if energy is added, it is added only to the momentum part of it

Absolutely not. Part of it has to always quantize orthogonally for
electromagnetic equilibrium to be maintained.

> now no ne agrues that the more mo0mentum you add
> the trijectory of the particle will be more 'flat'
> now why do i say it works againt your logic?:
> while you detect a quark, is it with its orriginal energy
> or with additional energy ? - obviously with additional
> energy

It now depends on the relative velocity of the particle that
collides with the quark with respect to that of the latter.

Remember that to even penetrate a proton, an electron must
go at nearly the speed of light, while it is quite possible
that the quarks themselves are moving at velocities in the
same range in their confined local system.

It would be quite unsurprising that with respect to each other
they may be moving rather slowly as they actually colide, both
their velocities cancelling each other to possibly a large
extent.

The inelastic collisions observed definitely confirm that
a rather low mass is encountered by the projectile electrons.

Of course, some collisions must have been totally elestic,
if the velocities completely added to each other instead of
cancelling out to various degrees, but I think that most collisions
will lie within some absolute minimum, close to the mass of the
electron, to some absolute maximum that must have been estimated.

But since I do not personally have access to the actual data of the
experiments, I don't know the figure.

The average masses estimated seem to lie between the figures I
already gave again recently, taken from the European Physical Journal
of year 2000.

> now what is its original situation in the nucleid?
> is it with the additional energy that was used during its 'discovery'?
> no it is without that addition

The electron is not destroyed in the collision, but it obviously
communicates part of its own energy to the quark that it hits. But
it is easy to take that transfered energy into account by subtracting it.

> so will it be in the nuleid more massive or less masive- acording
> to *your concept*/
> in the nucleid it should be less massive
> now
> in the more massive situation it is only a few percent of the nucleid mass
> so what will be in its less masive situation? *even less than
> those few miserable percent*!

Think that the current estimated mass is the conclusion of I don't know
how many hundreds of experiments that have been carried out.

I think that they must not be very far from the real "rest mass" of the
quarks.


> now lets examine anothe rexample:
> lets take an Ione of an element
> and measure its mass
> now lets take the same element with say an Ione that is missing
> say 3 electrons
> now the questuion is : what will be the mass diffwerence
> between case no one
> and case no 2 with the 3 electrons missing
> will it be a difference of 3 electron mass
> or will it be a difference of 3 electrons *+* some addition
> of any relativistic mass (ie more that 3 electrons mass) ???

Electrons are so light that I doubt that "measuring" the mass of
an atom missing 3 with respect to "measuring" the same atom with
all of its electrons would indicate any difference.

But electrons do not go at relativistic velocities in atoms. The
move (if at all) at velocities well below relativistic.

For all practical purposes, you just subtract 0.511 MeV per
electron.

If you want to take their motion into account, you can go
so far as to subtract approximately 0.511003 MeV per electron

:-]

> i think the answer is clear - it is experimental facts.

Yes. This is experimental facts

Regards

André Michaud

Y.Porat

unread,
Jan 8, 2004, 3:20:58 AM1/8/04
to
s...@microtec.net (Andr? Michaud) wrote in message news:<562f286c.04010...@posting.google.com>...
> map...@012.net.il (Y.Porat) wrote in message ne
> > > > > > -------------------

>
>
> > > >
> > > > 1 see 'The third Porat Postulation'
> > > >
> > > > there is no 'relativistic mass' a shoking idea? once it will be
> > > > well undestood, suddenly many 'fathers' of that idea will pop up
>
> > >
> > > > now anyone ever took the movement of the electron, in its mass
> > > > calculation?
> > >
> >
>
> > i would say at first glance that your logic works against you.:
> > the more general equation of energy is :
> > (E^2)times LF = (mc^)^2 + (pc )^2
> > in words
> > it is a vector combination of two vectors *added perpendicularly*
> > ie the momentum vector is independant of the rest mass vector
>
> Absolute agreement.
----------
a good start ....
now please note you asked what is LF
it is Lorentz factor and it used to be ascociated to the mass
my inovation is that it should be ascociated only to energy side
and you cant find anything against it because i did it
with a ligitimate algebraic move!
please note another fact:
the mc^2 vector and the pc vectors are perpendicular!!
it is not just a mathematical fact it is a physical fact
now please think about what perpendicular vectors mean:
it means *they are independant of each other *
in simpler words: and increnment of one of them cannot add
anything to the other one
(the same ingenious idea of your French genious De Cartes
with his ingenius coordinate system:
those coordinates are independant, and increment in
say x direction ads nothing to the y direction etc etc.
so while you add momentum and velocity to a particle
you add nothing to the perpendicular vector even if you
wish hard to do it !!!.
ie addition of velocity does not add anything
to the rest mass
i was looking at the chart you brought above
it is axactly as i expaect:
the fast electrons have a 'flatter' trajectory' than the slow ones
and i explained it to you in answer to your
circular accelerator argument:
the fast electron has a flatter path not because it got more mass
but because it got more *velocity* (momentum)
just to remind you my example:
if a car is passing in front of you say from left to right
and you have to hit it say with a macine gun with a constant
rate of shooting;
in which case you have a better chance to hit it with more bullets-
in the case it moves say 10 kmh
or in the case it moves 200 kmh ?
the similat physical phenomenon in the accelerator or even
in the wilson chamber:
the faster the particle is - the flatter will be its path.
(EM action is similar to shooting bullets! just remember the
duality of EM waives)
now the main point to innovate is:
it is not flatter because of addition of mass but
addition of velocity as i explained with the moving car and bullets.
------------------


-------


>
> > (an addition of momentum doea not add to the rest mass
>
> If real energy is involved, there is no way for it not to
> seek internal electromagnetic equilibrium. This means that
> part of the added energy will be expressed as momentum and
> part of it will quantize orthogonally to maintain this
> equilibrium. That second part has no way other than to
> associate through antiparallel spin with the already

-----
dont think so
as i saied the two vectors are (even mathmatically perpendicular
isnt that even a mathematical prove ?)
and there is no way to add anything to a vector by another one
who is pwependicular to it!
---------
-----------


> orthogonally quantized energy that constitutes what we call
> the rest mass of the particle.
>
> That part of the added energy that quantizes so that equilibrium
> is maintained becomes measurable as added mass to the rest mass
> of the particle.

------
no way.
-------


>
>
> I do not know what your LF means, but the original formula empirically
> takes into account the automatic quantization of part of the added
> energy, at the general level. This is why it gives exacts results when
> velocities are calculated for any amount of added energy.

---------
the increment and exact calculated is as is
the interpretation of it is wrong:
you have the constant vector of rest mass
and another vector of increasing velocity
*the resultant (obliqe) vector is actually becoming bigger
but not because of the rest mass part but the momentum part*
i di\ont believe you have a way to prove that the mass part was increased
itis the momentum increase
momentum is mv mv isincreased but how can you tell that it was
the m part and not the V part? (while the V part is actually
obviously and tangibly was increased !!.
if you suggest that it is more difficuly to add velocity
i dont argue with that but .....
because 'the mass increased'??
i give you the counter interpretation
of our 'car and bullets'
-----------------
------------


>
> But I personally prefer the following form, which allows directly
> calculating velocities and is very easy to use:
>
> v = c(sqrt(1-(mc^2/(K+mc^2))^2)
>
> For example, if you induce 10 MeV of energy into an electron, you get
>
> v = c(sqrt(1-(0,511/(10+0,511))^2)
>
> v= c(,998817556)
>
> v=299437972 m/s very close to the speed of light
>
> It is easy to substitute any energy value

-------
if it is right (didnt folow it thoroughly0
there is no contradiction to my explanations.


>
> > if energy is added, it is added only to the momentum part of it
>
> Absolutely not. Part of it has to always quantize orthogonally for
> electromagnetic equilibrium to be maintained.

-------
see above
i boubt if youcan prove it and not just make statsments.
it is against the mathemathics thogh as you know me
i am not a blind fan of mathemathics..........
----------
-----


>
> > now no ne agrues that the more mo0mentum you add
> > the trijectory of the particle will be more 'flat'
> > now why do i say it works againt your logic?:
> > while you detect a quark, is it with its orriginal energy
> > or with additional energy ? - obviously with additional
> > energy
>
> It now depends on the relative velocity of the particle that
> collides with the quark with respect to that of the latter.
>
> Remember that to even penetrate a proton, an electron must
> go at nearly the speed of light, while it is quite possible
> that the quarks themselves are moving at velocities in the
> same range in their confined local system.

---------
actually Andre niethr me nor you nor anyone realy known whats
going on there...... it is aspeculations.
----------


>
> It would be quite unsurprising that with respect to each other
> they may be moving rather slowly as they actually colide, both
> their velocities cancelling each other to possibly a large
> extent.

-----------
speculations ....
-------


>
> The inelastic collisions observed definitely confirm that
> a rather low mass is encountered by the projectile electrons.

-----
it is acollision of something known with *something unknown completely*
my private guess is of the 'all electron theory'
ie most of matter (above electron) is composed of the 'electron
family' but in the 'zoom in' world it might get some surprising
properties. ie much smaller much dense much stiffer etc etc.
----------


>
> Of course, some collisions must have been totally elestic,
> if the velocities completely added to each other instead of
> cancelling out to various degrees, but I think that most collisions
> will lie within some absolute minimum, close to the mass of the
> electron, to some absolute maximum that must have been estimated.

> -----
as i saied : 'the all elecron theory' ......

> But since I do not personally have access to the actual data of the
> experiments, I don't know the figure.

------
well saied!
----------


>
> The average masses estimated seem to lie between the figures I
> already gave again recently, taken from the European Physical Journal
> of year 2000.

--------
thank you for that sa i told you -it was you who gave me the information
and therefore the guts to be so blant and revolutionary
now you 'back up' ??? go ahead and make the right conclusions....
------------


>
> > now what is its original situation in the nucleid?
> > is it with the additional energy that was used during its 'discovery'?
> > no it is without that addition
>
> The electron is not destroyed in the collision, but it obviously
> communicates part of its own energy to the quark that it hits. But
> it is easy to take that transfered energy into account by subtracting it.

> ---------
too comlicated ansd elusive.
------


> > so will it be in the nuleid more massive or less masive- acording
> > to *your concept*/
> > in the nucleid it should be less massive
> > now
> > in the more massive situation it is only a few percent of the nucleid mass
> > so what will be in its less masive situation? *even less than
> > those few miserable percent*!
>
> Think that the current estimated mass is the conclusion of I don't know
> how many hundreds of experiments that have been carried out.

----------
is it a 'rest mass' or it icludes relativistic mass ?
anyway it is far from one third of the nucleid.
-------


>
> I think that they must not be very far from the real "rest mass" of the
> quarks.

> -----------
ok as above
------

> > now lets examine anothe rexample:
> > lets take an Ione of an element
> > and measure its mass
> > now lets take the same element with say an Ione that is missing
> > say 3 electrons
> > now the questuion is : what will be the mass diffwerence
> > between case no one
> > and case no 2 with the 3 electrons missing
> > will it be a difference of 3 electron mass
> > or will it be a difference of 3 electrons *+* some addition
> > of any relativistic mass (ie more that 3 electrons mass) ???
>
> Electrons are so light that I doubt that "measuring" the mass of
> an atom missing 3 with respect to "measuring" the same atom with
> all of its electrons would indicate any difference.

----------
people always tell me that those measurments are very acurate.
anyway far too acurate inorder to detect 'relativistic mass'!
ie yess or no relativistic mass.....
-------


>
> But electrons do not go at relativistic velocities in atoms. The
> move (if at all) at velocities well below relativistic.

-------
how do you know how much relativistic or not
me personally i dont believe they are rotating the y are ....
vibrating .........
-----


>
> For all practical purposes, you just subtract 0.511 MeV per
> electron.

---------
Bingo.. no relativistic is taken in acount.
if you dont mind -- 8that should be the conclusion:
intrinsic movement is not taken in mass calculations
or mass evaluations
-------------
------


>
> If you want to take their motion into account, you can go
> so far as to subtract approximately 0.511003 MeV per electron
>
> :-]

------
agreed
---------


>
> > i think the answer is clear - it is experimental facts.
>
> Yes. This is experimental facts

-------
yes
-------
Regards
Y.Porat
---------------
ps as shown above people can argue and still be friends......
---------------------
>
> Regards
>
> André Michaud

Michael Moroney

unread,
Jan 9, 2004, 3:54:12 PM1/9/04
to
map...@012.net.il (Y.Porat) writes:

>mor...@world.std.spaamtrap.com (Michael Moroney) wrote in message news:<btf5j9$28c$2...@pcls4.std.com>...
>>

>> First electron binding to an atom is a few eV, if you calculate the mass
>> of that energy it's very small.
>>
>> Second, if you want high precision you _would_ take into account the
>> motion energy.
>>
>> Consider also particle accelerators. At low energies, you can just plug
>> in the electron mass into the calculations to compute the voltages/
>> magnetic field strengths etc. to get the thing to work (such as an
>> electron gun in a TV). At very high energies that fails completely. The
>> electrons are moving at close to c, and are more massive than an at-rest
>> electron. Very much more for very high energies, so much so that the rest
>> mass of an electron is a tiny percentage of its total mass!
>> An electron with 10% of its mass its rest mass and the other 90% due to
>> its kinetic energy is _easily_ produced. It even happens naturally in
>> some beta decaying isotopes (5 MeV or more decay energy)
>----------------
>Hi Mike
>as i understand from Bjoern you got a 'copy of copy' of my book
>i hope a complete one wonder how ledgible it is that copy of copy.

Yes I have a photocopy with Bjoern's notes. It is quite legible although
Bjoern's writing is not so clear sometimes (but it's better than mine!)

>once you have an ione with one electron less than the 'ordinary'
>the differenc inmass will be just 'pure flat electron mass...'
>nothing else ie we measure it in its (not sure how it is called)
>its say rest position no additional energy is attached to it

The energy is there but when binding energies are only a few MeV, it
is a very small contributor to the total mass and is very hard to
measure.

--
-Mike

Y.Porat

unread,
Jan 10, 2004, 10:08:49 AM1/10/04
to
mor...@world.std.spaamtrap.com (Michael Moroney) wrote in message news:<btn4dk$5si$3...@pcls4.std.com>...

> map...@012.net.il (Y.Porat) writes:
>
> >mor...@world.std.spaamtrap.com (Michael Moroney) wrote in message news:<btf5j9$28c$2...@pcls4.std.com>...
> >>
> >> First electron binding to an atom is a few eV, if you calculate the mass
> >> of that energy it's very small.
> >>
> >> Second, if you want high precision you _would_ take into account the
> >> motion energy.
> >> beta decaying isotopes (5 MeV or more decay energy)
> >----------------
> >Hi Mike
> >as i understand from Bjoern you got a 'copy of copy' of my book
> >i hope a complete one wonder how ledgible it is that copy of copy.
>
> Yes I have a photocopy with Bjoern's notes. It is quite legible although
> Bjoern's writing is not so clear sometimes (but it's better than mine!)
> ---------------
there are some of mine notes in hanwriting
i hope you will notice it
but of course there is alot more to add to it
folowing my additional experience during last 10 years
from the orriginal book
if you wish (and i preffere it we can discuss it privatlty
with my private e mail which is now
por...@012.net.il
----------------

> >once you have an ione with one electron less than the 'ordinary'
> >the differenc inmass will be just 'pure flat electron mass...'
> >nothing else ie we measure it in its (not sure how it is called)
> >its say rest position no additional energy is attached to it
>
> The energy is there but when binding energies are only a few MeV, it
> is a very small contributor to the total mass and is very hard to
> measure.
--------------
my main point there is that while you eveluate mass
you dont take in acount internal movement of the subparticles.
because at least another reason- that is the natural situation
of the particle.
-------------
all the best
Y.Porat
-------------

Andr? Michaud

unread,
Jan 10, 2004, 4:00:22 PM1/10/04
to
map...@012.net.il (Y.Porat) wrote in message news:<4e35159f.04010...@posting.google.com>...

> s...@microtec.net (Andr? Michaud) wrote in message news:<562f286c.04010...@posting.google.com>...
> > map...@012.net.il (Y.Porat) wrote in message ne
> > > > > > > -------------------
> >
> >
> > > > >
> > > > > 1 see 'The third Porat Postulation'
> > > > >
> > > > > there is no 'relativistic mass' a shoking idea? once it will be
> > > > > well undestood, suddenly many 'fathers' of that idea will pop up
>
>
> > > > > now anyone ever took the movement of the electron, in its mass
> > > > > calculation?
> > > >
> > >
>
> > > i would say at first glance that your logic works against you.:
> > > the more general equation of energy is :
> > > (E^2)times LF = (mc^)^2 + (pc )^2
> > > in words
> > > it is a vector combination of two vectors *added perpendicularly*
> > > ie the momentum vector is independant of the rest mass vector
> >
> > Absolute agreement.
> ----------
> a good start ....
> now please note you asked what is LF
> it is Lorentz factor and it used to be ascociated to the mass
> my inovation is that it should be ascociated only to energy side
> and you cant find anything against it because i did it
> with a ligitimate algebraic move!
> please note another fact:
> the mc^2 vector and the pc vectors are perpendicular!!

Yes, but only part of the "pc". The orthodox formula is only approximate
in this regard. Part of it must quantize orthogonally if it is real
electromagnetic energy. See below.

> it is not just a mathematical fact it is a physical fact

Absolutely.

> now please think about what perpendicular vectors mean:
> it means *they are independant of each other *
> in simpler words: and increnment of one of them cannot add
> anything to the other one

This is where something seems to be missing in you logic.

Vectorial product is key to electromagnetism.

The electrical force acting on a charge is perpendicular
to the vectorial product of velocity by magnetic force, which
are themselves both perpendicular to each other.

Any basic electromagnetism textbook should document this.

> (the same ingenious idea of your French genious De Cartes
> with his ingenius coordinate system:
> those coordinates are independant, and increment in
> say x direction ads nothing to the y direction etc etc.

Not in electromagnetism. Here, vectorial product always
involves three perpendicular axis.

> so while you add momentum and velocity to a particle
> you add nothing to the perpendicular vector even if you
> wish hard to do it !!!.

Quite the contrary. If you verify this, you will see
that in electromagnetism this is the case. All electromagnetic
energy is subjected to that fundamental rule.

Orthogonal translation of energy between velocity in space,
electric force and magnetic force has been well understood
since the middle of the 19th century.

> ie addition of velocity does not add anything
> to the rest mass

Actually, you are right. What is added is not strictly speaking
"added mass", but rather "added inertia", the same kind of transverse
inertia that photons have. The part of the "pc" that quantizes orthognally
is measurable as added transverse inertia. The formula simply is too
general to clarify this aspect.

> i was looking at the chart you brought above
> it is axactly as i expaect:
> the fast electrons have a 'flatter' trajectory' than the slow ones
> and i explained it to you in answer to your
> circular accelerator argument:
> the fast electron has a flatter path not because it got more mass
> but because it got more *velocity* (momentum)

Definitely. But relativistic speeds are not studied in Wilson chambers.
This is for the wide range of non relativistic speeds.

For relativistic speeds, it is easier in circular accelerators.

There, at relativistic speeds the trajectories tend to become
flatter on account of more velocity PLUS more transverse inertia.

Both factors add to each other and both have to be taken into
account.

> just to remind you my example:


> if a car is passing in front of you say from left to right
> and you have to hit it say with a macine gun with a constant
> rate of shooting;
> in which case you have a better chance to hit it with more bullets-
> in the case it moves say 10 kmh
> or in the case it moves 200 kmh ?
> the similat physical phenomenon in the accelerator or even
> in the wilson chamber:
> the faster the particle is

...and the more transverse inertia it has...

> - the flatter will be its path.
> (EM action is similar to shooting bullets! just remember the
> duality of EM waives)

Yes. See above. In electromagnetism, velocity, magnetic force
and electric force are orthogonal to each other and are inter
dependent, meaning that energh can flow from one to the other
whenever equilibrium is broken.

> now the main point to innovate is:
> it is not flatter because of addition of mass but
> addition of velocity as i explained with the moving car and bullets.
> ------------------
>
>
> -------
> >
> > > (an addition of momentum doea not add to the rest mass
> >
> > If real energy is involved, there is no way for it not to
> > seek internal electromagnetic equilibrium. This means that
> > part of the added energy will be expressed as momentum and
> > part of it will quantize orthogonally to maintain this
> > equilibrium. That second part has no way other than to
> > associate through antiparallel spin with the already
> -----
> dont think so
> as i saied the two vectors are (even mathmatically perpendicular
> isnt that even a mathematical prove ?)

See vectorial product in electromagnetism.

> and there is no way to add anything to a vector by another one
> who is pwependicular to it!

Yes this is what is normal in electromagnetism.

> ---------
> -----------
> > orthogonally quantized energy that constitutes what we call
> > the rest mass of the particle.
> >
> > That part of the added energy that quantizes so that equilibrium
> > is maintained becomes measurable as added mass to the rest mass
> > of the particle.
> ------
> no way.
> -------
> >
> >
> > I do not know what your LF means, but the original formula empirically
> > takes into account the automatic quantization of part of the added
> > energy, at the general level. This is why it gives exacts results when
> > velocities are calculated for any amount of added energy.
> ---------
> the increment and exact calculated is as is
> the interpretation of it is wrong:
> you have the constant vector of rest mass
> and another vector of increasing velocity

Think that the rest mass of any particle is also made up of
electromagnetic energy.

> *the resultant (obliqe) vector is actually becoming bigger
> but not because of the rest mass part but the momentum part*
> i di\ont believe you have a way to prove that the mass part was increased
> itis the momentum increase
> momentum is mv mv isincreased but how can you tell that it was
> the m part and not the V part? (while the V part is actually
> obviously and tangibly was increased !!.
> if you suggest that it is more difficuly to add velocity
> i dont argue with that but .....
> because 'the mass increased'??
> i give you the counter interpretation
> of our 'car and bullets'

I understand what you are explaining to me, but what you say
means applying classical mechanics logic to electromagnetic
particles. In electromagnetism, classical mechanics can only
give a rough approximation of what happens.

> -----------------
> ------------
> >
> > But I personally prefer the following form, which allows directly
> > calculating velocities and is very easy to use:
> >
> > v = c(sqrt(1-(mc^2/(K+mc^2))^2)
> >
> > For example, if you induce 10 MeV of energy into an electron, you get
> >
> > v = c(sqrt(1-(0,511/(10+0,511))^2)
> >
> > v= c(,998817556)
> >
> > v=299437972 m/s very close to the speed of light
> >
> > It is easy to substitute any energy value
> -------
> if it is right (didnt folow it thoroughly0
> there is no contradiction to my explanations.
> >
> > > if energy is added, it is added only to the momentum part of it
> >
> > Absolutely not. Part of it has to always quantize orthogonally for
> > electromagnetic equilibrium to be maintained.
> -------
> see above
> i boubt if youcan prove it and not just make statsments.
> it is against the mathemathics thogh as you know me
> i am not a blind fan of mathemathics..........

Prove it is difficult, unless you have a look at the explanations
of the triple orthogonality of electrommagnetism in any basic
electricity and magnetism text book, and appreciate the proof
for yourself.

> ----------
> -----
> >
> > > now no ne agrues that the more mo0mentum you add
> > > the trijectory of the particle will be more 'flat'
> > > now why do i say it works againt your logic?:
> > > while you detect a quark, is it with its orriginal energy
> > > or with additional energy ? - obviously with additional
> > > energy
> >
> > It now depends on the relative velocity of the particle that
> > collides with the quark with respect to that of the latter.
> >
> > Remember that to even penetrate a proton, an electron must
> > go at nearly the speed of light, while it is quite possible
> > that the quarks themselves are moving at velocities in the
> > same range in their confined local system.
> ---------
> actually Andre niethr me nor you nor anyone realy known whats
> going on there...... it is aspeculations.
> ----------

I personally trust that these experiments have really been carried
out and that the scientists have honestly reported their findings
on the scattering experiments that have been posted for decades in
the European Physical Journal.

As for interpretation, I did my own interpretation based on the
sum of what I know.

But I was not revolutionary. This data has been public knowledge for
decades. All physicists are aware of them. They just don't discuss
it much, or then, only reluctantly, because they don't know how to
reconcile it with QFT.

These experimentally verified mass ranges of quarks up and down are
accepted as true experimental results by all physicists, even they
have no explanation.

> now you 'back up' ??? go ahead and make the right conclusions....
> ------------

Back up ? I don't understand what you mean by this.

> >
> > > now what is its original situation in the nucleid?
> > > is it with the additional energy that was used during its 'discovery'?
> > > no it is without that addition
> >
> > The electron is not destroyed in the collision, but it obviously
> > communicates part of its own energy to the quark that it hits. But
> > it is easy to take that transfered energy into account by subtracting it.
> > ---------
> too comlicated ansd elusive.
> ------
> > > so will it be in the nuleid more massive or less masive- acording
> > > to *your concept*/
> > > in the nucleid it should be less massive
> > > now
> > > in the more massive situation it is only a few percent of the nucleid mass
> > > so what will be in its less masive situation? *even less than
> > > those few miserable percent*!
> >
> > Think that the current estimated mass is the conclusion of I don't know
> > how many hundreds of experiments that have been carried out.
> ----------
> is it a 'rest mass' or it icludes relativistic mass ?
> anyway it is far from one third of the nucleid.

Nobody knows yet the actual rest mass of quarks up and down. The range
I gave you are an estimate drawn from the numerous scattering experiments
that were carried out.

So, as long as we have not really ascertained their real rest mass, these
estimates will of course include some relativistic mass (I would prefer
the term "relativistic inertia").

> -------
> >
> > I think that they must not be very far from the real "rest mass" of the
> > quarks.
> > -----------
> ok as above
> ------
> > > now lets examine anothe rexample:
> > > lets take an Ione of an element
> > > and measure its mass
> > > now lets take the same element with say an Ione that is missing
> > > say 3 electrons
> > > now the questuion is : what will be the mass diffwerence
> > > between case no one
> > > and case no 2 with the 3 electrons missing
> > > will it be a difference of 3 electron mass
> > > or will it be a difference of 3 electrons *+* some addition
> > > of any relativistic mass (ie more that 3 electrons mass) ???
> >
> > Electrons are so light that I doubt that "measuring" the mass of
> > an atom missing 3 with respect to "measuring" the same atom with
> > all of its electrons would indicate any difference.
> ----------
> people always tell me that those measurments are very acurate.

Yes.

> anyway far too acurate inorder to detect 'relativistic mass'!
> ie yess or no relativistic mass.....
> -------

For any particle that moves, there is a part of relativistic mass (I would
prever "relativistic inertia"), but for low velocities, it is so small
that it must be neglected, or else we would have to needlessly deal with
numbers with too many digits after the decimal.

> >
> > But electrons do not go at relativistic velocities in atoms. The
> > move (if at all) at velocities well below relativistic.
> -------
> how do you know how much relativistic or not
> me personally i dont believe they are rotating the y are ....
> vibrating .........
> -----

Ok. If they are vibrating instead of moving, then they certainly
cannot go at relativistic speeds.

> >
> > For all practical purposes, you just subtract 0.511 MeV per
> > electron.
> ---------
> Bingo.. no relativistic is taken in acount.

Ok. I if you want to take it into account, you would get
a figure of this type: 0.5110015 MeV

Judge for yourself, which is more reasonable to use.

What they call relativistic velocities simply is the higher
velocity range where the added inertia become significant enough
to take into account in calculation when it actually makes a
difference.
With all lower velocities, the relativistic quantities involved are
too cumbersome and insignificant to bother with.

> if you dont mind -- 8that should be the conclusion:
> intrinsic movement is not taken in mass calculations
> or mass evaluations
> -------------
> ------
> >
> > If you want to take their motion into account, you can go
> > so far as to subtract approximately 0.511003 MeV per electron
> >
> > :-]
> ------
> agreed
> ---------
> >
> > > i think the answer is clear - it is experimental facts.
> >
> > Yes. This is experimental facts
> -------
> yes
> -------
> Regards
> Y.Porat
> ---------------
> ps as shown above people can argue and still be friends......
> ---------------------

Absolutely.

Regards

André Michaud

Y.Porat

unread,
Jan 11, 2004, 1:18:19 AM1/11/04
to
s...@microtec.net (Andr? Michaud) wrote in message news:<562f286c.04011...@posting.google.com>...

> map...@012.net.il (Y.Porat) wrote in message news:<4e35159f.04010...@posting.google.com>...
> > s...@microtec.net (Andr? Michaud) wrote in message news:<562f286c.04010...@posting.google.com>...
> > > map...@012.net.il (Y.Porat) wrote in message ne
> > > > > > > > -------------------
> > >
> > >
> > > > > >
> > > > > > 1 see 'The third Porat Postulation'
> > > > > >
> > > > > > there is no 'relativistic mass' a shoking idea? once it will be
> > > > > > well undestood, suddenly many 'fathers' of that idea will pop up
> > ---------------
Hi andre
btw the apostrof you add to your name becomes in our poor asci
a question mark)
i with your permission snipp all the rest of that stagnant
discussion
and invite you to join my thread:
'The third Porat Postulate'
because all of our above 9or below) discussion is concentrated
in that formula that i introduces there in the first article

just a remerk to your EM argument to say that forces
can be added to perpendicular axis
it might be right in EM do you know why?
because there is in nature a sub basic particle that moves
naturally in a closed circle! (my other postulate)
once it moves in a circle- it does not move just along one axis!!!.
that is another physics (you see mathemathics alone is not good enough
we have to know the real physical facts- wich we dont!!!
2 what makes rest mass we dont know as well!!
but if there is something there (which i believe)
that already moves with the speed of light (that does not
contradict my assumption of vibrations you can find a model that
combines all of that !!) than you can add nothing
to something that go to its *maximum* rest mass is something that
got to its maximum!!
-------
so please read it and try to join in
Regrds
Y.porat
--------------
>

Andr? Michaud

unread,
Jan 11, 2004, 4:17:52 PM1/11/04
to
map...@012.net.il (Y.Porat) wrote in message news:<4e35159f.04011...@posting.google.com>...

> s...@microtec.net (Andr? Michaud) wrote in message news:<562f286c.04011...@posting.google.com>...
> > map...@012.net.il (Y.Porat) wrote in message news:<4e35159f.04010...@posting.google.com>...
> > > s...@microtec.net (Andr? Michaud) wrote in message news:<562f286c.04010...@posting.google.com>...
> > > > map...@012.net.il (Y.Porat) wrote in message ne
> > > > > > > > > -------------------
> > > >
> > > >
> > > > > > >
> > > > > > > 1 see 'The third Porat Postulation'
> > > > > > >
> > > > > > > there is no 'relativistic mass' a shoking idea? once it will be
> > > > > > > well undestood, suddenly many 'fathers' of that idea will pop up
> > > ---------------
> Hi andre
> btw the apostrof you add to your name becomes in our poor asci
> a question mark)

Yes I know. Unfortunately, I am unable to change that setting that
dates back to my original login to google.

I should have entered Andre but instead entered André.

I am stuck with it.

> i with your permission snipp all the rest of that stagnant
> discussion
> and invite you to join my thread:
> 'The third Porat Postulate'
> because all of our above 9or below) discussion is concentrated
> in that formula that i introduces there in the first article

> just a remerk to your EM argument to say that forces
> can be added to perpendicular axis
> it might be right in EM do you know why?

Nobody knows why. We only know that it is objective physical
reality.

For example, electric motors work because of it. One set up is
providing excess electrical force in a constant perpendicular
magnetic field.

Electromagnetic equilibrium being broken, and the magnetic force
being fixed (thus unable to absorb any of the excess energy), that
excess energy has no option but to flow by orthogonal translation
perpendicular to both forces and cause the rotor to turn.

If left to itself, electromagnetic energy always falls into perfect
equilibrium.

The speed of light, for example is the equilibrium speed of free
electromagnetic energy (photons).

Half of a photons energy (which could metaphorically be represented
by the vectorial product of its electric aspect by its magnetic aspect,
both perpendicular to each other) is sort of pushed at the speed of
light by the other half which is acting in space perpendicular to
both electric and magnetic aspects.

You have equilibrium whenever you have as much energy in electric
plus magnetic aspect on one hand, and normal space on the other
hand.

This is why massive particle cannot reach the speed of light.
They are forever overburdened on the electromagnetic side.

> because there is in nature a sub basic particle that moves
> naturally in a closed circle! (my other postulate)
> once it moves in a circle- it does not move just along one axis!!!.
> that is another physics (you see mathemathics alone is not good enough
> we have to know the real physical facts- wich we dont!!!

We do for electromagnetic energy, and all stable massive particles.

> 2 what makes rest mass we dont know as well!!

We positively know that it is made up of electromagnetic energy.

> but if there is something there (which i believe)
> that already moves with the speed of light (that does not
> contradict my assumption of vibrations you can find a model that
> combines all of that !!) than you can add nothing
> to something that go to its *maximum* rest mass is something that
> got to its maximum!!
> -------
> so please read it and try to join in

Thank you for the invitation.

But I never was hot to enter any all out discussion. I prefer to
occasionally provide info or references to individuals who seem
to need or want it or maybe discuss a little with interested
individuals.

Regards

André Michaud

Y.Porat

unread,
Jan 12, 2004, 12:05:47 AM1/12/04
to
s...@microtec.net (Andr? Michaud) wrote in message news:<562f286c.04011...@posting.google.com>...
> map...@012.net.il (Y.Porat) wrote in message new
--------------------
sory for another top post:
you seems to preffere to go 'on safe ground'
ie to quote only 'proven things'
which is no doubt a legitimate way
antway i am a bit more an adventurer than you because....
i whant some advance, the existing situation is at least for me
far from satisfying.
so once we take say EM you are right that magnetic field
and electric field are perpendicular
now the difference between us seems to be that i am not
happy with that explanation as deep enough
we all know that EM is actually *one * phenomena it is not two
the division to two is just a human artifact for practical
handling or more correst because .....
*we didnt find yet **something closer to reality***
so in my devious character i am looking for deeper
and i start to guess..
so my curent guess is that an EM wave is actually
some unknown particle that moves a two fold way:
1 in a closed circle
2 in addition to 1 in a straght line perpendicular to the
above crecle so what do we get althogether?-
a hellix movement!
i preffere it because it at least looks simpler
and i dont know why , i believe that basic phenomena of nature
are much more simpler than our complicated and 'sophysticatse' minds
if not simple enough it whant last so long.
IOW it seems that we lack some more effective tools to describe
natural phenomena, sometimes our mathematics is not good enough
to do the job effectively.
another bug of our scince is ....
smugness! peaple are too smug with the existing situation
and give too much credit to demagogues.
-------
Regards
Y.Porat
--------------

Andr? Michaud

unread,
Jan 12, 2004, 12:52:06 PM1/12/04
to
map...@012.net.il (Y.Porat) wrote in message news:<4e35159f.0401...@posting.google.com>...

> s...@microtec.net (Andr? Michaud) wrote in message news:<562f286c.04011...@posting.google.com>...
> > map...@012.net.il (Y.Porat) wrote in message new
> --------------------
> sory for another top post:
> you seems to preffere to go 'on safe ground'
> ie to quote only 'proven things'

"Proven things" are the only things that we know for certain
about objective reality. They are the only foundation possible
to enventually understand completely that objective reality.

This is why I readily inform young minds about them when I see
one looking for information.

> which is no doubt a legitimate way

The only way, believe me.

> antway i am a bit more an adventurer than you because....
> i whant some advance, the existing situation is at least for me
> far from satisfying.

I think that you have no idea how unconventional I am by quoting
only "proven things". Most physicists regard this as "suspicious".

> so once we take say EM you are right that magnetic field
> and electric field are perpendicular
> now the difference between us seems to be that i am not
> happy with that explanation as deep enough

I never was either. I actually went much deeper, but on public
forums, it is not a good idea to go deeper than quoting "proven
things". When you do, you instantly get blasted by self appointed
guardians of orthodoxy. Not very constructive.

I chose instead to spread my views directly into schools, at all
levels, for students to get access to them before they make up
their mind about orthodox theories.

> we all know that EM is actually *one * phenomena it is not two
> the division to two is just a human artifact for practical
> handling or more correst because .....

Well, it is not exactly a human artifact, but you are absolutely
right: electric aspect and magnetic aspect of EM energy are only that,
2 aspects of the same fundamental thing, fundamental quantities of
motion induced by acceleration.

> *we didnt find yet **something closer to reality***
> so in my devious character i am looking for deeper
> and i start to guess..

I already told you that you have a good mind, and that you
are on the right path. You just need more information to
go all the way.

All the necessary "proven things" have already been understood.
You only need to identify them in the sea of literature and put
them together to see the whole come into perspective. You will
need however to reconsider the basic space geometry if you want
to succeed.

> so my curent guess is that an EM wave is actually
> some unknown particle that moves a two fold way:
> 1 in a closed circle
> 2 in addition to 1 in a straght line perpendicular to the
> above crecle so what do we get althogether?-
> a hellix movement!
> i preffere it because it at least looks simpler

You are close, but there is yet a simpler solution.

> and i dont know why , i believe that basic phenomena of nature
> are much more simpler than our complicated and 'sophysticatse' minds
> if not simple enough it whant last so long.

Total agreement.

> IOW it seems that we lack some more effective tools to describe
> natural phenomena, sometimes our mathematics is not good enough
> to do the job effectively.

Total agreement. But before being able to describe, mathematically
or otherwise, one must understand what we want to describe. In other
words, one must have "something" to describe.

A hint: To succeed in the direction you want, you must do away with
Minkowski's 4 dimentional space geometry. It does not properly
describe fundamental reality.

> another bug of our scince is ....
> smugness! peaple are too smug with the existing situation
> and give too much credit to demagogues.

Total agreement.

Maybe you will like this little informatory text that I put up publicly
a few years back.

http://pages.globetrotter.net/srp/socroyan.htm

Regards

André Michaud

Y.Porat

unread,
Jan 13, 2004, 2:31:03 AM1/13/04
to
s...@microtec.net (Andr? Michaud) wrote in message news:<562f286c.04011...@posting.google.com>...
> map...@012.net.il (Y.Porat) wrote in message news
> > sory for another top post:
> > you seems to preffere to go 'on safe ground'
> > ie to quote only 'proven things'
>
>
> This is why I readily inform young minds about them when I see
> one looking for information.
>
> > which is no doubt a legitimate way
>
> The only way, believe me.
-----------------
i agree that you have to inform young people with
'the best state of art'
yet imho that is not enough:
young people (not all of them!)tend to take anything delivered to them
by 'an authority' as 'complete truth'
my teaches at secondary scool always told us :
'dont be a parrot- use your own mind'!!
sory but i took it to seriously .......
that 'parrot' word that i use many times against members here
is not my fault... it is my teachers fault...... (;-)
we have to inform them as well the amount of uncertainty
that invoves the 'state of art information'
that is vitally important if we whant real advance!
do you know , personaly while i first learned about modern
physics-
the 'pose' that scintists and teachers was so self confident
that i had no doubt in my mind, that everything is under control
and the r is actually nothing to add or contribute.
and let me tell you that while i was developing my Model-
can you guess what was tho most difficult part of it ???
you will never guess......
it was
to get rid of some 'solid paradigma' !!!
it costed me a horrible waist of time and effort!
for instance - the shell model and idea that a heavy atom has
the same number of electrons as the Protons etc .
while scintists are making funny junglairing tricks to
stuff all of them actually as they wish
sometimes a shell is uuper in another case on of its electrons
goes inwards.. why ... because..... it didnt fit
althogether a big mess! and every one is swolowing that garbage
nonsense without blinking an eye!
-------------
once i got reed of it..
anything started at once 'to run ahead'
that is just a little example.
-----------------

>
> > antway i am a bit more an adventurer than you because....
> > i whant some advance, the existing situation is at least for me
> > far from satisfying.
>
> I think that you have no idea how unconventional I am by quoting
> only "proven things". Most physicists regard this as "suspicious".
--------
thats the way
to use the existing- but with sucpicion
------------

>
> > so once we take say EM you are right that magnetic field
> > and electric field are perpendicular
> > now the difference between us seems to be that i am not
> > happy with that explanation as deep enough
>
> I never was either. I actually went much deeper, but on public
> forums, it is not a good idea to go deeper than quoting "proven
> things". When you do, you instantly get blasted by self appointed
> guardians of orthodoxy. Not very constructive.
>
> I chose instead to spread my views directly into schools, at all
> levels, for students to get access to them before they make up
> their mind about orthodox theories.
-------
ok but to add the uncertainty element.

>
> > we all know that EM is actually *one * phenomena it is not two
> > the division to two is just a human artifact for practical
> > handling or more correst because .....
>
> Well, it is not exactly a human artifact, but you are absolutely
> right: electric aspect and magnetic aspect of EM energy are only that,
> 2 aspects of the same fundamental thing, fundamental quantities of
> motion induced by acceleration.
>
> > *we didnt find yet **something closer to reality***
> > so in my devious character i am looking for deeper
> > and i start to guess..
>
> I already told you that you have a good mind, and that you
> are on the right path. You just need more information to
> go all the way.
>
> All the necessary "proven things" have already been understood.
> You only need to identify them in the sea of literature and put
> them together to see the whole come into perspective. You will
> need however to reconsider the basic space geometry if you want
> to succeed.
------
most of it is aleady there but still a lot of need of
'recombination of information'
see fpr instance my thread ' the third Porat postulate'
how about by just moving the gama factor from one side
of the e=mc^2 times gama factor
from the right side to the left side sudeenly .......
*rest mass disappeared*!!!!!
its amazing!
-------------

>
> > so my curent guess is that an EM wave is actually
> > some unknown particle that moves a two fold way:
> > 1 in a closed circle
> > 2 in addition to 1 in a straght line perpendicular to the
> > above crecle so what do we get althogether?-
> > a hellix movement!
> > i preffere it because it at least looks simpler
>
> You are close, but there is yet a simpler solution.
---------
Andre just do a favoutr to me and try to remember my
'basic particle that moves naturally in a closed circle'
a temporary name ' The Circlon' i dont need curved space' !!!
------------

>
> > and i dont know why , i believe that basic phenomena of nature
> > are much more simpler than our complicated and 'sophysticatse' minds
> > if not simple enough it whant last so long.
>
> Total agreement.
>
> > IOW it seems that we lack some more effective tools to describe
> > natural phenomena, sometimes our mathematics is not good enough
> > to do the job effectively.
>
> Total agreement. But before being able to describe, mathematically
> or otherwise, one must understand what we want to describe. In other
> words, one must have "something" to describe.
>
> A hint: To succeed in the direction you want, you must do away with
> Minkowski's 4 dimentional space geometry. It does not properly
> describe fundamental reality.
---------
4 dimentions is nice but my 'Circlon' is nicer!
(remember out beliefe in simplicity!! it is much simpler.
-------

>
> > another bug of our scince is ....
> > smugness! peaple are too smug with the existing situation
> > and give too much credit to demagogues.
>
> Total agreement.
>
> Maybe you will like this little informatory text that I put up publicly
> a few years back.
>
> http://pages.globetrotter.net/srp/socroyan.htm
-----------
thanks i enterd it to my archive.
-----
>
> Regards
>
> André Michaud
----------
Regards
Y.porat
-----------------

Andr? Michaud

unread,
Jan 13, 2004, 8:46:57 AM1/13/04
to
map...@012.net.il (Y.Porat) wrote in message news:<4e35159f.04011...@posting.google.com>...

> s...@microtec.net (Andr? Michaud) wrote in message news:<562f286c.04011...@posting.google.com>...
> > map...@012.net.il (Y.Porat) wrote in message news
> > > sory for another top post:
> > > you seems to preffere to go 'on safe ground'
> > > ie to quote only 'proven things'
> >
> >
> > This is why I readily inform young minds about them when I see
> > one looking for information.
> >
> > > which is no doubt a legitimate way
> >
> > The only way, believe me.
> -----------------
> i agree that you have to inform young people with
> 'the best state of art'
> yet imho that is not enough:

No it is not enough. The rest has to be done by the
readers themselves. No one can force horses to drink.

What I do is bring water to them. The rest is out of my
control.

But on the other hand, it is also impossible that no one
will be thirsty, and out of the thousands of copies already
circulated, it is impossible that all of them will be thrown
away or ignored.

When I am completely done, more than 10 thousand copies
will have been distributed. Books tend to last and usually
end up in libraries.

It is sufficient that a few of them be read, the mental
effort invested to really comprehend the model, for it to
eventually be understood and begin to really be discussed.

Some day progress will resume.

> young people (not all of them!)tend to take anything delivered
> to them by 'an authority' as 'complete truth'

Yes. This also works both ways.

It is what they understand first that counts. They tend to
doubt anything that afterwards seems in contradiction with
what they first accepted.

That's the key, and I have been turning it for 3 years now.

> my teaches at secondary scool always told us :
> 'dont be a parrot- use your own mind'!!
> sory but i took it to seriously .......
> that 'parrot' word that i use many times against members here
> is not my fault... it is my teachers fault...... (;-)
> we have to inform them as well the amount of uncertainty
> that invoves the 'state of art information'
> that is vitally important if we whant real advance!
> do you know , personaly while i first learned about modern
> physics-
> the 'pose' that scintists and teachers was so self confident
> that i had no doubt in my mind, that everything is under control
> and the r is actually nothing to add or contribute.
> and let me tell you that while i was developing my Model-
> can you guess what was tho most difficult part of it ???
> you will never guess......
> it was
> to get rid of some 'solid paradigma' !!!
> it costed me a horrible waist of time and effort!

I agree. It is the most difficult part. It is exactly why no
currently practicing physicist can induce change. It is mentally
impossible for any of them to requestion the cricical paradigms.

Progress can come only from the next generation, and then, only
from those individual who will have laid hand on the proper
information before they set their minds to orthodox paradigms.
This is why I am not wasting my time trying to explain anything
to the current crop of physicists.

> for instance - the shell model and idea that a heavy atom has
> the same number of electrons as the Protons etc .
> while scintists are making funny junglairing tricks to
> stuff all of them actually as they wish
> sometimes a shell is uuper in another case on of its electrons
> goes inwards.. why ... because..... it didnt fit
> althogether a big mess! and every one is swolowing that garbage
> nonsense without blinking an eye!
> -------------
> once i got reed of it..
> anything started at once 'to run ahead'
> that is just a little example.
> -----------------
> >
> > > antway i am a bit more an adventurer than you because....
> > > i whant some advance, the existing situation is at least for me
> > > far from satisfying.
> >
> > I think that you have no idea how unconventional I am by quoting
> > only "proven things". Most physicists regard this as "suspicious".
> --------
> thats the way
> to use the existing- but with sucpicion

The "proven things" are in a special class. Once you have ascertained
how they can really be proven, you can let go of any suspiscion about
them.

> ------------
> >
> > > so once we take say EM you are right that magnetic field
> > > and electric field are perpendicular
> > > now the difference between us seems to be that i am not
> > > happy with that explanation as deep enough
> >
> > I never was either. I actually went much deeper, but on public
> > forums, it is not a good idea to go deeper than quoting "proven
> > things". When you do, you instantly get blasted by self appointed
> > guardians of orthodoxy. Not very constructive.
> >
> > I chose instead to spread my views directly into schools, at all
> > levels, for students to get access to them before they make up
> > their mind about orthodox theories.
> -------
> ok but to add the uncertainty element.

Not about the "proven things" that we talked about. They are the
only solid foundation that can be really built on.

> >
> > > we all know that EM is actually *one * phenomena it is not two
> > > the division to two is just a human artifact for practical
> > > handling or more correst because .....
> >
> > Well, it is not exactly a human artifact, but you are absolutely
> > right: electric aspect and magnetic aspect of EM energy are only that,
> > 2 aspects of the same fundamental thing, fundamental quantities of
> > motion induced by acceleration.
> >
> > > *we didnt find yet **something closer to reality***
> > > so in my devious character i am looking for deeper
> > > and i start to guess..
> >
> > I already told you that you have a good mind, and that you
> > are on the right path. You just need more information to
> > go all the way.
> >
> > All the necessary "proven things" have already been understood.
> > You only need to identify them in the sea of literature and put
> > them together to see the whole come into perspective. You will
> > need however to reconsider the basic space geometry if you want
> > to succeed.
> ------
> most of it is aleady there but still a lot of need of
> 'recombination of information'

Yes.

> see fpr instance my thread ' the third Porat postulate'
> how about by just moving the gama factor from one side
> of the e=mc^2 times gama factor
> from the right side to the left side sudeenly .......
> *rest mass disappeared*!!!!!
> its amazing!

Well, the E already contains the energy represented by the gamma
to start with.

E already contains all of the energy expressed by the
expression located on the other side of the equal sign.

But mathematically, of course all transformations are allowed.
But they are useful only if they can serve some purpose.

I find this equation useful if you want to find the
total energy, or the mass, or the p, or the velocity of
a particle if you decompose the gamma. That last option
is the one I showed you a few posts back.

> > > so my curent guess is that an EM wave is actually
> > > some unknown particle that moves a two fold way:
> > > 1 in a closed circle
> > > 2 in addition to 1 in a straght line perpendicular to the
> > > above crecle so what do we get althogether?-
> > > a hellix movement!
> > > i preffere it because it at least looks simpler
> >
> > You are close, but there is yet a simpler solution.
> ---------
> Andre just do a favoutr to me and try to remember my
> 'basic particle that moves naturally in a closed circle'
> a temporary name ' The Circlon' i dont need curved space' !!!

Yes. A very nice idea. In flat space, but still in a 4 dimentional
set up.

> ------------
> >
> > > and i dont know why , i believe that basic phenomena of nature
> > > are much more simpler than our complicated and 'sophysticatse' minds
> > > if not simple enough it whant last so long.
> >
> > Total agreement.
> >
> > > IOW it seems that we lack some more effective tools to describe
> > > natural phenomena, sometimes our mathematics is not good enough
> > > to do the job effectively.
> >
> > Total agreement. But before being able to describe, mathematically
> > or otherwise, one must understand what we want to describe. In other
> > words, one must have "something" to describe.
> >
> > A hint: To succeed in the direction you want, you must do away with
> > Minkowski's 4 dimentional space geometry. It does not properly
> > describe fundamental reality.
> ---------
> 4 dimentions is nice but my 'Circlon' is nicer!
> (remember out beliefe in simplicity!! it is much simpler.

As far as I understand your very interesting circlon, it still lives
in a 4 dimentional space.

Think of what you said above about how tough it was for you
to mentally let go of some paradigms.

The same effort is ahead about this. And this is the toughest.

Electromagnetism is the key.

Regards

André Michaud

0 new messages